NBCOT STUDY BUNDLE PACK SOLUTION (Questions and Answers )(2022/2023) (Verified Answers)

Does spina bifida include dysfunction with hands?
no
-usually LE: gait difficulties, bowel and bladder problems

Dystonia
movement disorder in which your muscles contract involuntarily, causing repetitive or twisting movements
-can be focal (one part of body), segmental (2 or more), general (all of body)

Prader-Willi Syndrome
-genetic disorder
-small hands and feet, abnormal growth and body composition (small stature, very low lean body mass, and early-onset childhood obesity), hypotonia (weak muscles) at birth, insatiable hunger, and intellectual disability

Reactive Attachment Disorder Behaviors
-high need to be in control
-frequent lying
-affectionate and overly related w/strangers
-frequent episodes of hoarding or gorging on food
-denial of responsibility
-projecting blame for their actions on others

Postural Interventions to Increase Swallowing

  1. chin tuck
  2. forward head tilt
  3. head turn

What does pocketing of food imply?
-sensory impairment of oral cavity
-lack of awareness of residual food

Low contrast sensitivity
-problems with night driving, including difficulty seeing pedestrians walking alongside poorly lit streets
-eyes tire more easily while reading or watching television.

Macular Degeneration
-deterioration of the central portion of the retina
-central vision loss
-controls ability to read, drive a car, recognize faces or colors, and see objects in fine detail.

Anosognosia
lack of insight, lack of awareness

What does the posterior branch of brachial plexus include?
radial nerve
-wrist drop dysfunction

Bulbar tract involvement w/ ALS
difficulty breathing, slurred speech, decreased volume of speech, or impaired swallowing

Optical character recognition (OCR)
enables you to convert different types of documents, such as scanned paper documents, PDF files or images captured by a digital camera into editable and searchable data

Errorless Learning
-client is prompted to make correct response immediately, ensuring a correct response each time
-prompt is slowly faded in order to promote accuracy with least amount of errors and frustration

Autonomic Activation
-increase in the activity of the autonomic nervous system
-physical system responsible for nonconsciously maintaining bodily homeostasis and coordinating bodily responses

Backward chaining
-procedure that begins with the last element in the chain and proceeds to the first element
-if there are 4 steps to an action, therapist does first 3 and client does last…(then only 2, etc..)

What syndrome often presents w/ food-seeking behavior?
Pradi-Willi syndrome

What technique should be used w/ CRPS to help gain function back?
stress loading activities

Pyschodynamic FOR
-discussion based
-suggests unresolved childhood events are reason for dysfunction

CBT
-thoughts and reactions
-can use journaling, observational learning and modeling skills

Cognitive Disability FOR
-uses clients strength to allow for function
-intervention could include training caregivers to provide appropriate environmental support

Behavioral FOR
-relies on idea that behavior is learn and that it can be unlearned
-breathing and relaxation techniques can facilitate change in response
-use repetition to shape behavior

What are the signs of dementia w/ Lewy bodies?
visual hallucinations and Parkinson-like motor symptoms

What is the first phase of sensory reeducation
-instruction in visual compensatory strategies
-concerned w/ safety d/t total sensory loss

Veracity
-be honest w/ claims of ability and professional skills
-must have timely, accurate documentation

Can Ayres Sensory Integration intervention be applied in groups?
no, only individually

Bradydactyly
-short digits
-could cause difficulty with FM tasks of ADL

Can a person w/ low 5 ACL participate in social interactions?
-have difficulty w/ this because they are egocentric

What is an appropriate intervention for lower-extremity burns to help decrease pain in standing?
provide vascular support before standing to decrease blood pooling

What is TRICARE?
health coverage for military

Can an entry-level or novice OT do a Functional Capacity Evaluation?
No

What sling can be used to promote neutral wrist position for handwriting?
rubber band sling

Confrontation testing
assesses how much peripheral vision client has

Amsler grid
assesses central visual fields

Can an OT give a client an optical device?
no, it has to be prescribed by optometrist or ophthalmologist

Position of long arm splint for cubital tunnel syndrome
45-60 degrees flexion to minimize stretch of ulnar nerve

Can a COTA administer and score a Mini-Mental State Exam?
yes, however they need minimal supervision to periodically recheck service competency

Prinicpal of motivation interviewing
avoid argument and opposition to client’s resistance
-aka: roll with resistance

What is useful in helping clients initiate movement and minimize effects of bradykinesia?
self-cueing

What is a sign of cranial nerve injury and oculomotor dysfunction?
double side by side images, pupils are asymmetrical, and ptosis
-should be evaluated

Transitional Work Programs
-monitor client progress
-reassess performance to determine when job tasks can be upgraded to full time

What ACL is appropriate for volunteer activities?
5.6

Interoception
ability to perceive information from inside the body
-ex: children w/ autism have difficult time with toileting

Intervention for juvenile RA
-A/PROM
-splinting
-monitoring joint function
-prevent deformation
-teach energy conservation techniques
-use of AE

What does congenital syphilis result in?
osteochondritis at joints
-PROM will be painful

Achondroplasia
dwarfism

Sensorimotor approach to handwriting
-variety of sensory experiences
-different media
-novel instruction

Premotor perservation
repetition of task that already has been done
-ex: keep pulling up a sock that has been put on already

What area of the hand is 2-point discrimination post refined?
volar fingertips

What is the best compensatory cognitive strategy for a client with MS?
schedule demanding tasks at intervals through the day

Print Tool
standardized assessment to measure child’s ability to produce handwriting

How do you minimize graft rejection?
maintain immobilization for 2-7 days

What is the best pressure garment to prevent hypertrophic scaring
-Jobst pressure garment
-wounds have to be closed

How many days after anticoagulation can therapy be administered?
2-3 days

How can you help patient w/ PD in initiation of performance of ADL
use external cues and repetition

Postprandial Orthostatic Hypotension
-occurs when person goes from supine to standing shortly after eating a meal
-affects older adults, high BP, and PD

What color is skin w/ chronic bronchitis?
blues skin and lips w/ edema

What color is skin w/ emphysema?
pink

Does hot water make it easier or harder for patient w/ COPD to breath?
harder
(humidity makes it difficult)

What MET level is a client at in cardiac care unit?
1-2 METS

Symptoms of L sided heart failure
anxiety, difficulty breathing, cerebral hypoxia

Symptoms of R sided heart failure
cyanotic nail beds, jaundice, and LE edema

What do Type III fractures of radial head require?
removal of fragmented bone and cast for 3-4 weeks

What does Zone IV and V of flexor tendons include?
median nerve

Nerve Testing Order
-one point moving
-one point discrimination
-2 point moving
-2 point discrimination

What FOR is good to base an eval for a child with ODD?
CBT

What does the Children’s Health Insurance Program Cover?
children and families whose incomes are too high for Medicaid, but too low for private insurance

What should caregivers do during freezing episodes in clients w/ PD
external cues and feedback

What are cross out assessment used for?
-visual scanning
-detecting spatial neglect of visual field impairments

Early sign of ALS
-distal
-weakness of small muscles of hand is most typical

What does ideational practice involve?
knowing which objects are used and how to use them

First step in eccentric view
increasing clients awareness of blindspot

Signs of Neurofibromatosis in Children
-short stature
-skeletal anomalies

If a COTA changes the population they are working with, what type of supervision is needed?
close

Strong Magnification
-there is a small field of vision
-magnifier should be held to eye

How do you get rid of a boney block?
surgically

Recognition
ability to note key features and relate them to what is already kept in memory

Matching
ability to note similarities among objects

Sorting
-done by type and size
-ability to categorize objects on basis of similarities and note differences

What does Donepezil (Aricept) reduce/side effect?
reduce: negative mood, anxiety, hallucination
side effect: dizziness

Antidepressent risk factors
suicide, mania, and hypomania

How many standard deviations are needed for services?
higher than 1.5

What is Milwaukee brace used for?
children w/ kyphosis or posterior convexity

Good proprioceptive activity for kids
-climbing a rope
-has good input through muscles and joints

What stage of AD are motor skill deficits noticed?
late stage AD

What is an important component of Recovery Model?
peer support and teaching

How do you initially remove pressure off ischial tuberosity’s in a wheelchair?
remove footrests

How many months before you can do a re-evaluation that insurance will cover?
6 mo

Do footrests need to be measured for wheelchairs?
no
-seat width, depth, height, and seat-back height are the 4 measurements required

What is the suggested interrater reliability of a standardized assessment?
80%

What is the most common psychiatric disorder seen in organ transplant patients?
anxiety

What visual perception causes mixing up b’s and d’s
visual discrimination

Contextual factors
temporal, personal, cultural, virtual

Reisburg’s stages for dementia
-stages 1 and 2
-stage 1: no disability noted
-stage 2: person complains about forgetting normal age-related information (location of keys, wallet, etc.)

Reisburg’s stages for dementia
-stage 3
-independent in IADL, uses compensation techniques
-forgets important information, has difficulty completing complex tasks, difficulty with directions to new locations

Reisburg’s stages for dementia
-stage 4
-can perform simple and repetitive ADL, lives at home with support, follows simple verbal cues
-increasingly forgetful, unable to follow and sequence written cues, unable to perform familiar activities that are challenging, difficulty in word finding, needs assistance to manage at home

Reisburg’s stages for dementia
-stage 5
-ADL and some IADL with correct cues and assistance, responds to encouragement, unable to safely drive
-poor judgment, difficulty with all decision making, forgets to take care of hygiene

Reisburg’s stages for dementia
-stage 6
-can complete familiar tasks and follow demonstration/hand over hand cues
-difficult time following 2-step task, cannot sequence steps of ADLs, cannot speak in full sentences, incontinent

Middle stage AD characteristic
clients’ thinking that they are back in an earlier stage of life

Claw hand
-seen in ulnar nerve problems
-third and fourth lumbricals are innervated by the ulnar nerve
-there is a hyperextension of the MCP of ring and little finger

Histrionic Personality Disorder
-colorful, dramatic, extroverted behavior
-inability to maintain deep, long-lasting attachments
-flamboyant presentation

Conversion Disorder
condition in which you show psychological stress in physical ways

What is the goal of an injury prevention program?
minimize employer expenses related to lost time and wages

Who are considered primary referral sources for FCE?
physicians, case managers, and attorneys

What is an appropriate functional training phase activity for someone using a unilateral myoelectric TD?
-using scissors to cut paper
-the TD assists to hold the paper

Who would be appropriate for the use of a knife with a bent handle?
-someone with RA and ulnar drift
-it keeps wrist and hand in neutral

What is barrel chest a sign of?
emphysema

What assessment can be used if MOHO FOR is used?
role checklist

Feedforward praxis
enables person to compare previous motor plans and sensory info to detect potential errors and correct the plan before attempting again

Executional praxis
ability to perform motor action w/precision

What are examples of visual cognitive functions?
visual memory and figure ground

What type of visual skills deficit is affected by CNS damage?
stereopsis

What can help break up extensor tone when person is sitting?
positioning hips into 90 degree or more flexion

What deficits can a person with kyphosis have?
eating skills

Who would be appropriate to use eye tracking input?
lack of head control and upper extremity movement

Who would be appropriate for switch encoding?
someone who has ability to input with at least one finger

Scaffolding
working with client on components of activity that are difficult, but client ultimately finishes activity independently

What FOR articulates 3 dimensions of doing?
-MOHO
-participation, performance, and skills level
-top down approach

Leg lifter
-device for moving one leg at a time
-helps manually move legs over side of bed or left legs into bed

Thoracic Outlet Syndrome
-compression, injury, or irritation of nerve and/or blood vessel in lower neck and upper chest
-usually affects athletes that do repetitive arm or shoulder movements
-neck, shoulder, arm pain, and numbness or impaired circulation of affected areas

Useful Field of View
-assess ability to scan info using central and peripheral vision
-process info in timely manner

Dexmethylfenidate (Focalin) side affect
moodiness and agitation
-used for ADHD

An OT is preparing to evaluate a toddler who has upper extremity orthopedic concerns. How will the OT MOST likely obtain the majority of initial assessment data?

A. Measurement tools that assess visual-motor skills
B. Dynamometer and pinch meter readings.
C. Observation of child during activities in the child-care center.
D. Functional independence measures.
C. Observation of child during activities in the child-care center.

Through observation of the child during child-care center activities, the OT can collect information about the child’s motor performance skills and participation in activities that require upper extremity/hand skill. Naturalistic observation is a method of ecological assessment, which is “a primary mechanism for obtaining data relevant to the child’s performance context….. Skilled observation of child performing a functional task offers….. important information about the child’s performance”. Answers A, B, ad D are all appropriate choices after the child is old enough for these assessments.

During an initial evaluation, the OT suspects that a child has somatodyspraxia. In what area should the OT focus the evaluation?

A. Ability to print or write.
B. Reading competency.
C. Math calculations.
D. New motor task planning.
D. New motor task planning.

“Somatodyspraxia is described as a deficit in learning new motor skills, planning new motor actions, and generalizing motor plans.” Inability to print or write (answer A) is termed “dysgraphia.” The term “dyslexia” (answer B) means dysfunction in reading. Inability to perform mathematics (answer C) is known as “dyscalculia”

An OT working in a long-term care facility needs to evaluate the long-term memory of a resident. Which of the following methods is BEST for evaluating memory or personally experienced events (declarative memory)?

A. Show the person a series of objects and ask him to recall the objects within 60 seconds.
B. Ask the individual how he spent New Year’s/
C. Have the individual state the place, date, and time.
D. Ask the client to remember to bring a specific item to the next therapy session
B Ask the individual how he spent New Year’s

“Declarative memory is one aspect of long term memory and includes conscious memory for events, knowledge or facts”. It is commonly assessed through verbal interviews and informal testing such as asking a question about an individual’s recall of personal events (answer B). Working memory refers to “the temporary storage of inforamtion while one is working with it or attending to it” (answer A). “Prospective memory involves the ability to remember intentions or activities that will be required in the future” (answer D). Knowing the date, place, and time is indicative of orientation (answer C).

A child avoids playground equipment that requires her feet to be off the ground. What does this behavior MOST likely indicate?

A. Difficulty modulating proprioception.
B. Somatodyspraxia
C. Gravitational insecurity
D. Bilateral integration/sequencing deficit.
C. Gravitational insecurity.

Gravitational insecurity is described as “fear response to movement”. The child easily experiences a fear of falling and prefers to keep her feet firmly on the ground. Tactile defensiveness (answer A) is a term used to describe discomfort with various textures and with unexpected touch. Somatodyspraxia (answer B) has is “foundation in somatosensory (e.g. primarily tactile but also proprioceptive) discrimination deficits, which interfere with the development of body scheme and awareness”. Bilateral integration and sequencing deficits are related to “poor vestibular-propioceptive discrimination, which interferes with the ability to coordinate, sequence, and execute motor actions quickly and efficiently”.

When the OT suspects tactile defensiveness as a rationale for a child’s challenges, in what area of participation should the OT focus on FIRST?

A. Play behavior
B. Dressing habits
C. Social skills
D. Leisure interests
B. Dressing habits

Children with tactile defensiveness are “bothered by tactile aspects of daily living activities…. specific types of clothing…. specific textures materials”. The child may be bothered by certain textures or avoid wearing turtlenecks, socks, or shoes. Conversely, some children may never take off their shoes to avoid tactile stimulation. Play behavior (answer A), social skills (answer C), and the choice of hobbies (answer D) could be affected secondarily, as a result of intolerance to certain textures or human touch. Knowledge of the child’s dressing habits will give the OT key information at the start of the evaluation process.

An OT is working with an individual with schizophrenia who is in the process of preparing to move from a state hospital to a group home. During a baking group, the client becomes agitated and leaves the room when another client uses the electric hand mixer to mix the cake batter, and again when two clients begin to argue loudly about which type of icing to use. How would the OT BEST describe the behavior?

A. Low registration
B. Sensory avoiding
C. Sensation seeking
D. A hearing impairment
B. Sensory avoiding

The individual’s actions are indicative of sensor avoiding behavior, characterized by a low threshold to stimuli perceived as noxious, followed by an active response such as leaving the room. Individuals with sensory avoiding behavior may “become distressed in situations in which they cannot control the environment” and “do well in low stimulus situation or settings that others find dull”. An individual with low registration (answer A), sensory seeking behavior (answer C), or a hearing impairment (answer D) would not have difficult with the auditory stimulation caused by the roar of the mixer or loud voices.

During a self-care evaluation of an individual who recently sustained a brain injury, the OT instructs the individual to comb his hair immediately after he washes his face. The individual washes his face quickly, but then the therapist must give him several reminders to comb his hair. The OT is MOST likely to identify this as a deficit in what area?

A. Working memory
B. Judgment
C. Hearing
D. Abstraction
A. Working memory

“Working memory is the temporary storage of information while one is working with it or attending to it. It includes the ability to recall information immediately after exposure. It allows one to focus conscious attention and keeps track of information as one is performing an activity”. This individual’s inability to comb is hair without reminders suggests a deficit in working memory (answer A). Judgment (answer B), the ability to make realistic and safe decisions based on available environmental information, would not be needed for this task. Because the person performed the first request, hearing (answer C) would seem to be intact. Abstraction (answer D) is the ability to extrapolate information from an idea to generalize to another situation and would not be needed to follow this direction.

A supermarket employee with obsessive-compulsive disorder takes an hour to stock 24 soup cans on the shelf because once he has placed the cans on the shelf, he removes them and starts over, stating that “all labels were not lines up exactly in the same direction.” Which of the following methods would MOST effectively evaluate the individual’s work performance?

A. On-site observation of performance skills
B. Formal cognitive assessment
C. Verbal interview focusing on the requirements of the job
D. Task evaluation using a “clean” medium such as a puzzle
A. On-site observation of performance skills

An OT has been working with an individual who is recovering from a TBI. A standard pivot transfer has been successfully demonstrated in the gym. The MOST appropriate way to assess generalization of this new learning would be to have the patient perform which activity?

A. Identify potential hazards in the patient’s bathrooms that could make transfers unsafe
B. Select an appropriate tub bench and nonskid mat for the patient’s bathroom at home
C. Attempt a standard pivot transfer from wheelchair to bed in the patient’s hospital room
D. Attempt a sliding board transfer from wheelchair to tub
C. Attempt a standard pivot transfer from wheelchair to bed in the patient’s hospital room

An OT is working with an individual with depression who is cognitively intact but demonstrating difficulty carrying out self-care and other ADL tasks. The OT, who has no advanced certifications, would like to identify a standardized assessment to measure ADL performance. Which is the MOST appropriate tool for this purpose?

A. Bay Area Functional Performance Evaluation
B. Routine Task Inventory-Expanded
C. Kohlman Evaluation of Living Skills
D. Assessment of Motor and Process Skills
C. Kohlman Evaluation of Living Skills

An OT is conducting a perceptual function screening with an individual who has had a CVA. Which of the following informal screening activities would the therapist ask the individual to perform in order to identify the presence of agnosia?

A. Demonstrate common gestures such as waving
B. Name objects through touch only
C. Identify or demonstrate the use of common household objects
D. Read a paragraph and explain its meaning.
C. Identify or demonstrate the use of common household objects

An individual with an L4 spinal cord injury wishes to become independent in driving an automobile. The MOST appropriate piece of adaptive equipment for this individual is:

A. A palmar cuff for the steering wheel
B. A spinner knob on the steering wheel
C. Pedal extensions for accelerating and braking
D. Hand controls for acceleration and braking
D. Hand controls for acceleration and braking

What is the MOST important aspect of administering and scoring a standardized test?

A. Judgment to determine how best to administer the test
B. Previous experience as a way to gauge test results
C. Adherence to specific instructions for administration and scoring
D. Practice administering the test items
C. Adherence to specific instructions for administration and scoring

During an initial interview, parents describe their child as having difficulty communicating and interacting with others. The OT observes him repeatedly gazing upward and scanning the ceiling or quickly patting his hip. The behaviors described are MOST likely to be associated with what disorder?

A. Attention deficit-hyperacitivity
B. Childhood conduct
C. Obsessive-compulsive
D. Autism spectrum disorder
D. Autism spectrum disorder

A toddler with spina bifida has been referred for assessment. When collecting the initial data during interview with the child’s parent, what should the OT focus on PRIMARILY?

A. A parent’s concerns and goals for the child
B. Child’s medical management
C. Equipment needs
D. Physical layout of the home
A. A parent’s concerns and goals for the child

When evaluating an individual with coronary artery disease for controllable risk factors, what is MOST important for the OT to include as part of the assessment?

A. Determine the individual’s age and gender
B. Assess the individual’s lifestyle and dietary habits
C. Observe the individual for obesity and cholesterol levels
D. Determine whether the individual has a family history of heart disease
B. Assess the individual’s lifestyle and dietary habits

The OT has been working with an individual with deficits in the area of executive functioning postsurgery. When the OT asks about scheduling therapy appointments after discharge and transportation to the outpatient clinic, the client is unable to identify how to obtain the clinic’s phone number or where to park. When the OT points out these missing areas, the client appears perplexed, then laughs it off, stating that someone else would be calling to schedule appointments and providing transportation. Attempting to subtly hint about cognitive deficits is no longer working, and the OT is concerned because despite repeated efforts the individual is not retaining safety precautions. This patient’s behavior indicates a problem in what area?

A. Denial
B. Self-awareness
C. Emotional regulation
D. Sequencing
B. Self-awareness

An OT is evaluating two-point discrimination in an individual with median nerve injury. What is the BEST way to administer this test?

A. Apply the stimuli beginning at the little finger and progress toward the thumb
B. Start with the thumb area first, then progress toward the little finger
C. Present stimuli in an organized pattern to improve reliability during retesting
D. Allow the individual unlimited time to respond
A. Apply the stimuli beginning at the little finger and progress toward the thumb

An OT is initiating an evaluation of a pre-school child diagnosed with autism spectrum disorder. What is the OT MOST likely to include in the evaluation process?

A. One-to-one interview with the child
B. Observation of the child in a social, gross motor, and self-feeding task
C. The Peabody Developmental Motor Scales-2
D. Assessment of the child’s performance skills while outside on the playground
B. Observation of the child in a social, gross motor, and self-feeding task

A high school teacher diagnosed with a right-hemisphere CVA is given a paper with letters of the alphabet displayed in random order across the page and is instructed to cross out every “M.” The individual misses half of the “M”s in a random pattern. What type of deficit would cause such a response?

A. A left visual field cut
B. A right visual field cut
C. Functional illiteracy
D. Decreased attention
D. Decreased attention

A college student with a history of substance abuse has been admitted to the hospital following an accidental overdose at a party. He states his goal is to return to school as soon as possible so that his GPA does not drop below 3.0. What is the MOST important area for the OT evaluation to focus on?

A. Leisure skills
B. Activities of daily living
C. Academic/study skills
D. Family education
A. Leisure skills

What are the MOST important items to assess when evaluating motor control after a traumatic injury?

A. Developmental factors and primitive reflexes
B. Muscle tone, postural tone, reflexes, and coordination
C. Blood pressure, heart rate, endurance, and confusion
D. Self-concept and self- awareness
B. Muscle tone, postural tone, reflexes, and coordination

A new parent recently returned to work and reports difficulty concentrating at work because of thoughts about the baby. When at home, the individual feels distracted by thoughts about work. This MOST likely suggests the need to help this individual in what area?

A. Parenting skills
B. Attention span
C. Assertiveness
D. Role performance
D. Role performance

An OT is beginning to work with individuals in recovery in a community-based program. The OT needs an evaluation tool that can be administered in less than 20 minutes, identifies occupational performance limitations can be used to establish goals based on client priorities, and measure outcomes. Which is the BEST tool for this purpose?

A. Occupational Performance History Interview II
B. Role checklist
C. Canadian Occupational Performance Measure (COPM)
D. Occupational Self-Assessment
D. Occupational Self-Assessment

While making brownies, an individual is able to obtain all the supplies from the cabinet and check the oven temperature periodically. However, when the TV is turned on halfway through the activity, she becomes involved with the program and burns the brownies. This individual is showing signs of a deficit in which area?

A. Sustained attention
B. Detecting and reacting
C. Shifting of attention
D. Mental tracking
D. Mental tracking

In preparation for an annual Individualized Family Service Plan (IFSP) meeting, the OT is developing ideas for intervention to develop a child’s fine motor performance for independent play participation. To match the principles of Part C Early Intervention programs. What would be the OT’s BEST recommendation?

A. Family encouragement of the child to engage in play with toys he has at home, without concern for normal development
B. A periodic reassessment using Peabody Developmental Motor Scales, 2nd edition, fine motor subtest to measure progress in fine motor skill achievement
C. Home visit, during which OT will identify and encourage opportunities for fine motor practice within the family’s preferred routines
D. Increased weekly home visits so OT can implement therapy activities to promote child’s fine motor skill development, using a remedial approach
C. Home visit, during which OT will identify and encourage opportunities for fine motor practice within the family’s preferred routines

An adolescent with a history of shoplifting and gang violence has been hospitalized with a diagnosis of conduct disorder. During a task group, what are the MOST important performance skills for the OT to evaluate?

A. Perceptual-motor performance
B. Leisure and vocational interests
C. Attention span and social interaction skills
D. Interest in performing and ability to perform multiple roles
C. Attention span and social interaction skills

An OT is evaluating an individual who has undergone a total hip replacement to determine awareness and adherence to hip precautions prior to discharge. What can the OT conclude when the individual is observed leaning forward and stopping at 90 degrees of flexion to use the long-handled shoehorn while donning shoes?

A. Demonstrates independence with precautions
B. Requires verbal cuing to observe precautions
C. Needs a longer assistive device
D. Demonstrates cognitive deficits
A. Demonstrates independence with precautions

A client sustained a hand injury months ago while at work. The individual is now diagnosed with complex regional pain syndrome and is experiencing pain that interferes with work and self- care. Which of the following would be the best pain management modality option to discuss with the client?

A. Hot packs
B. Cold packs
C. Fluidotherapy
D. Transcutaneous electrical nerve stimulation
D. Transcutaneous electrical nerve stimulation

A child has considerable difficulty with problem-solving when playing with interlocking blocks, becomes frustrated, and gives up easily. An OT would MOST likely suspect a problem in what area?

A. Sensorimotor play
B. Pretend play
C. Constructional play
D. Play with rules
C. Constructional play

The OT is making recommendations to a community living site for a 13-year-old child with moderate intellectual disabilities. Which statement MOST accurately describes the functional ability of this child?

A. The child requires nursing care for basic survival skills
B. The child can usually handle routine daily functions in a supervised home
C. The child requires supervision to accomplish productive, nonroutine work.
D. The child is able to learn academic skills at the third- to seventh-grade level
B. The child can usually handle routine daily functions in a supervised home

An individual with borderline personality disorder has been referred to occupational therapy. Which of the following would be MOST important to evaluate?

A. Activities of daily living
B. Instrumental ADLs
C. Interpersonal skills
D. Sensorimotor skills
C. Interpersonal skills

An OT performing a motor skills evaluation observes that a child is awkward at many gross motor tasks. Though able to skip rope forward, the child is unable to skip rope backward, even after several attempts. This information would lead the therapist to be particularly observant for which additional signs?

A. Delayed reflex integration
B. Inadequate bilateral coordination
C. Developmental dyspraxia
D. General incoordintion
C. Developmental dyspraxia

A preteen with spastic cerebral palsy wants to use a computer. Prior to evaluating computer needs, what should the OT learn about FIRST?

A. The student’s and family’s goals for the device use
B. The family’s ability to afford a computer and its upgrades
C. Computer learning programs to facilitate student’s participation in the classroom
D. The student’s physical and cognitive capacities to determine appropriate keyboard and screen options
A. The student’s and family’s goals for the device use

An OT has been asked to develop a stress management program for clients with eating disorders. For this population in particular, which is the MOST important strategy to include?

A. A physical activity such as yoga or tai chi
B. Diaphragmatic breathing or progressive relaxation techniques
C. Training to acknowledge feeling and express emotions
D. Role-playing stressful situations
C. Training to acknowledge feeling and express emotions

The office doorway of an individual using a wheelchair has a clear opening of 28 inches. According to ADA guidelines, which of the following recommendations would be the MOST appropriate to facilitate clear passage of the wheelchair through the doorway?

A. The doorway width needs to be expanded to have a minimum clearance of 32 inches
B. The client needs to obtain a wheelchair narrower than 28 inches
C. The doorway width needs to be expanded to have a minimum clear opening of 45 inches
D. The doorway width is satisfactory and needs no modification
A. The doorway width needs to be expanded to have a minimum clearance of 32 inches

An individual is about to be discharged from outpatient OT after rehabilitation for a hand injury. The individual has not been able to work for 3 months and is unable to perform all of the job requirements as a truck driver. Which of the following should the OT practitioner recommend at discharge?

A. A home exercise program
B. Home health OT
C. Work hardening
D. No further OT services
C. Work hardening

An older adult has been hospitalized with a diagnosis of major depressive disorder following an overdose of sleeping pills. In obtaining an occupational profile, the OT discovers the patient has knitted and crocheted and enjoyed cooking. What activity should the OT practitioner recommend FIRST to achieve the goals of increasing a sense of confidence?

A. Crocheting a sweater for her teenage granddaughter
B. Making spaghetti and garlic bread for her husband
C. Knitting a small hat for her newborn grandson
D. Planning meals for the week in anticipation of discharge
C. Knitting a small hat for her newborn grandson

A child demonstrates aggressive and disruptive behavior in school as result of a low sensory threshold. Which suggestions would be MOST useful to discuss with the teacher regarding an upcoming class bus trip to the zoo?

A. Review the bus rules with the child and apply consequences consistently
B. Seat the child at the from of the bus and use earmuffs to dampen noise
C. Have child monitor classmates as “bus patrol” and report behavior
D. Let the child set the criteria for a successful trip, and provide a reward if the criteria are met
B. Seat the child at the from of the bus and use earmuffs to dampen noise

As the school team meets to develop the IEP for a third-grade student with autism spectrum disorder, the OT helps to project longer-term academic and functional performance outcomes for the student in middle school and high school. What is the MOST significant benefit of this approach for program planning?

A. Assurance that services will be available for the student in future years
B. Likelihood for optimal progress toward relevant post-high school goals
C. Current services that focus on steps to achieve targeted long-range outcomes
D. Avoidance of unnecessary duplication of services from year to year
C. Current services that focus on steps to achieve targeted long-range outcomes

A preschooler is having difficulty performing tasks requiring eye-hand coordination as a result of poor visual tracking skills. What activity should the OT use FIRST to promote visual tracking skills?

A. Tossing and catching a water balloon
B. Catching and bursting soap bubbles
C. Throwing and catching a beach ball
D. Playing softball
B. Catching and bursting soap bubbles

A homemaker with weak grip strength wishes to prepare a muffin mix but cannot open the bag. Which of the following would the OT MOST likely recommend?

A. A hand-powered mixer
B. Looped handle scissors
C. An electric knife
D. Prepare slice cookies instead of muffins
B. Looped handle scissors

An individual’s family wants to build a ramp to the primary entrance of the home. What is the maximum slope that the OT should recommend to the family?

A. 1 inch of ramp for every foot of ride in height
B. 1 foot of ramp for every inch of rise in height
C. 10 inches of ramp for every 2 inches in height
D. 1 foot of ramp for every foot of rise in height
B. 1 foot of ramp for every inch of rise in height

In establishing long-term goals for an individual with complete T4 paraplegia in a rehabilitation setting, the OT would MOST likely predict that the patient will attain what level of independence with bathing, dressing, and transfers?

A. Complete independence with self-care and modified independence with transfers
B. Independence with self-care and minimal assistance with transfers
C. Minimal assistance with self-care and moderate assistance with transfers
D. Dependence with both self-care and tranfers
A. Complete independence with self-care and modified independence with transfers

When planning acute treatment for a patient who has recently experienced a traumatic amputation of his right upper extremity at the below-elbow level, which of the following areas of patient education would the OT address FIRST?

A. Teaching to put on and take off a prosthesis
B. Training in residual limb wrapping
C. Practicing grasp and prehension functions
D. Simulation to resume vocational activities
B. Training in residual limb wrapping

An OT is working with an individual who is experiencing a manic episode and is highly excitable. Given that this individual has expressed interest in all kinds of craft activities, which type of craft activity would the OT be MOST likely to select to provide external structure for the client?

A. Doing a detailed needlepoint project requiring fine stitches
B. Using clay to shape an object of one’s choice
C. Doing a watercolor paint-by-numbers project
D. Finishing a prefabricated wood birdhouse from a kit
D. Finishing a prefabricated wood birdhouse from a kit

An OT in the hospital outpatient department meets with the parents of a 7-year-old boy with developmental coordination disorder (DCD). The OT believes his performance challenges his participation in school and brings this up. What is the MOST appropriate way for the OT to address the parents?

A. Request permission to send progress report to the school district
B. Ask parent for permission to call the school district’s OT to learn about the IEP referral process
C. Suggest that the school district pay for the child’s outpatient therapy, as it likely relates to the child’s school performance
D. Ask whether the school district has addressed his coordination difficulties and if not, discuss further whether the parent wishes to raise this issue with the child’s teacher
D. Ask whether the school district has addressed his coordination difficulties and if not, discuss further whether the parent wishes to raise this issue with the child’s teacher

An individual with complete C4 tetraplegia is able to independently use a mouth stick to strike keys on a computer keyboard for 3 minutes. To upgrade this activity, the OT practitioner should:

A. Provide a heavier mouth stick
B. have the individual work at the keyboard for 5 minutes
C. progress the individual to a typing device that inserts into a wrist support
D. Teach the individual how to correctly instruct a caregiver in use of the keyboard
B. have the individual work at the keyboard for 5 minutes

An individual who is considered modified independent for functional mobility consistently leaves her cane in another room. When asked where the cane is, the client replies, “Oh, that cane – it’s just so ugly.” Which of the following actions is MOST appropriate to take?

A. Discuss issues related to self-concept with the individual
B. Evaluate the individual’s short-term memory
C. Assess the individual’s long-term memory
D. Devise strategies to address time management
A. Discuss issues related to self-concept with the individual

What is the PRIMARY goal for providing a hand orthosis to a child with active juvenile rheumatoid arthritis?

A. Inhibit hypertonus
B. Increase range of motion
C. Prevent deformity
D. Correct deformity
C. Prevent deformity

Which of the following actions should the OT practitioner instruct a patient to perform FIRST when initiating a safe wheelchair transfer??

A. Have the patient scoot forward to the front of the seat
B. Position foot plates in the up position
C. Swing away the leg rests
D. Lock the brakes
D. Lock the brakes

An OT is treating a child with autism spectrum disorder. To maximize the child’s benefit from intervention using Ayres Sensory Integration, what does the therapist need to ensure?

A. Sensory strategies re provided to enable child to process proprioception and tactile stimuli
B. Only one option for activity is available at a time. to encourage focus
C. Active participation and self-direction in activities matched to identified needs
D. Highly structured session, facilitating step by step learning
C. Active participation and self-direction in activities matched to identified needs

An individual with ALS and mild dysphagia becomes extremely fatigued at meals. Which is the FIRST intervention the OT practitioner should consider recommending?

A. Speak with the physician about tube feeding
B. Sit in a semireclined position during meals
C. Eat six meals a day
D. Substitute pureed food for liquids.
C. Eat six meals a day

An OT has developed a work conditioning program for men and women who were previously homeless and exhibit generally decreased endurance. What is the FIRST part of a work conditioning program?

A. Work activities adapted to the level of their ability
B. Exercise and limited work task simulation
C. Work tasks specific to the jobs they will be getting
D. A full day of on-the-job training
B. Exercise and limited work task simulation

A sales executive being treated for anxiety is participating in OT to develop time management skills. Which of the following would be the expected outcome for the individual?

A. To control anxiety when arriving late for a meeting
B. To take responsibility when late with reports
C. To cope with feelings of inadequacy when missing a deadline
D. To eliminate late arrival work
D. To eliminate late arrival work

An individual has been instructed to place towels, one at a time, on a high shelf to improve shoulder function. The individual is able to easily place 10 towels. Which of the following modifications would MOST effectively improve endurance in the shoulder flexors?

A. Place the towels on a higher shelf
B. Increase the number of towels from 10 to 20
C. Place the towels on a lower shelf
D. Add a 1-pound weight to each arm
B. Increase the number of towels from 10 to 20

The OT is planning intervention for a 10-year-old child with learning disabilities and significant difficulties accomplishing writing and drawing tasks, secondary to perceptual-motor dysfunction. When selecting a service delivery approach, which is the BEST choice?

A. Consultation with classroom teach so student stays in least restrictive environment
B. 1:1 intervention from the OT to focus on student’s skill development before middle school
C. Intervention from the OT in a small group with other students having similar needs
D. A combination of service delivery models so that the OT can address student, teacher, and task needs
D. A combination of service delivery models so that the OT can address student, teacher, and task needs

A person with a long history of Parkinson’s disease is experiencing considerable fatigue during the day. The BEST way to enable the individual to maintain his level of function is to teach him how to:

A. work through the fatigue
B. perform desired activities in a simplified manner to conserve energy
C. employ pursed-lips breathing
D. eliminate activities or reduce activity level as much as possible
B. perform desired activities in a simplified manner to conserve energy

A child with behavioral problems has difficulty with peer interactions. The OT’s intervention plan is MOST likely based on which approach?

A. Provide small group occupation-based activities in an authoritarian environment with clear expectations
B. Begin with 1:1 occupation-based activities to develop child’s social skills, then introduce group activities
C. Provide small group occupation-based activities that encourage exploration and interaction
D. Provide small group occupation-based activities with rules to define acceptable play guidelines
C. Provide small group occupation-based activities that encourage exploration and interaction

An OT is developing a measurement plan to track changes in social participation of several students with autism spectrum disorder. Which is the MOST likely measurement strategy the OT selects?

A. School Function Assessment
B. Documentation of change in narrative progress notes
C. Photographs taken over a 2-month period during recess
D. Goal Attainment Scaling
D. Goal Attainment Scaling

An individual with Guillain-Barre acute syndrome demonstrates poor to fair strength throughout the upper extremities. Which is the most appropriate approach for the OT practitioner to use when planning treatment for the EARLY stages?

A. Gentle, nonresistive activities
B. Progressive resistive exercise
C. Fine motor activities
D. Active range of motion against moderate resistance
A. Gentle, nonresistive activities

A child with underreactive sensory processing has been referred to occupational therapy. Based on a sensory integration frame of reference, intervention activities should have which facilitatory characteristics?

A. Arrhythmic and unexpected
B. Arrhythmic and slow
C. Sustained and slow
D. Unexpected and rhythmic
A. Arrhythmic and unexpected

A child with cerebral palsy and limited postural stability is developmentally ready for toileting. Which element of the treatment plan should be considered FIRST?

A. training in management of fasteners
B. utilization of foot supports
C. provision of a seat belt
D. training in climbing onto the toilet
B. utilization of foot supports

The mother of a 3-year-old with spastic quadriplegia wants her son to walk independently around the house. The child is not yet ready to achieve this goal; however, his mobility would be helpful to the mother who is finding her child difficult to lift. What is the BEST way to assist in the development of family-centered intervention?

A. support and work on the parent’s goal ass she has stated it
B. suggest an alternate goal of improving sitting balance for playing
C. propose a modified goal that still meets the parent’s needs
D. include the child in the goal-setting process
C. propose a modified goal that still meets the parent’s needs

Following radiation therapy for breast cancer, an individual develops lymphedema in the right (dominant) upper extremity. She is experiencing functional deficits and is referred to occupational therapy. Treatment to reduce the swelling will begin with:

A. change of dominance training
B. RUE exercise using isometric contractions
C. fitting of compression garments
D. A heat modality, such as warm compresses
C. fitting of compression garments

An OT is planning intervention with a young teenage following second-degree skin burns on her hands and face. What other tool would add important information to what the therapist has already learned from the medical record, evaluation of child’s physical, and sensory function and ADL performance?

A. school record, including past grades and extracurricular activities
B. Canadian Occupational Performance Measure (COPM)
C. Jacob’s Prevocational Assessment (JPVA)
D. Ranchos Los Amigos Levels of Cognitive Functioning
B. Canadian Occupational Performance Measure (COPM)

An OT practitioner is designing a stress management series using a cognitive-behavioral treatment approach fro individuals who have chronic fatigue syndrome. What should the FIRST module in the series include?

A. Teaching time management techniques
B. identifying thoughts and beliefs that contribute to negative feelings
C. providing aerobic exercise
D. teaching how to perform progressive resistive exercise
B. identifying thoughts and beliefs that contribute to negative feelings

An OT is designing a series of group sessions for adolescents with eating disorders. What is the long-term goal for the group?

A. Develop healthy eating behaviors and meal preparation skills
B. Improve school performance
C. Develop independence in menu planning and awareness of portion size
D. Promote communication skills and assertiveness
A. Develop healthy eating behaviors and meal preparation skills

An individual with a complete high-level tetraplegia spinal cord injury is returning home. Which type of adaptive technology would the individual MOST likely require to ensure safety in the home?

A. a simple electronic aid to daily living (EADL)
B. a second generation EADL device with speakerphone
C. a remote control power door opener
D. an electric page turner
B. a second generation EADL device with speakerphone

What should an OT do to promote playfulness and self-expression in a young child with mild intellectual disability?

A. model imagination, and use playful expressions and voice
B. ask child to demonstrate his favorite things to do during playtime
C. provide child with toys that are familiar and played with frequently
D. provide activities with a means of release, such as leather tooling
A. model imagination, and use playful expressions and voice

As the OT plans intervention for a parent with a mild intellectual disability who has returned home from the NICU with her daughter who was born 6 weeks prematurely, the therapist considers ways o help this mother develop positive parenting skills during feeding and dressing activities. Which is MOST likely going to help this parent?

A. provide handouts that picture the sequence of steps required to prepare the baby’s formula and launder clothes in the washing machine
B. encourage problem-solving about how she will respond when the baby is fussy during feeding a diaper changes
C. consult with the service coordinator to ensure that caretaking supports are available for the mother
D. practice preparing formula and washing clothes and include multiple opportunities for repetition
B. encourage problem-solving about how she will respond when the baby is fussy during feeding a diaper changes

An OT is working with an individual with Alzheimer’s disease who demonstrates mild to moderate decline. She lives with her husband, who works during the day. The OT should initially focus intervention on which one of the following areas?

A. ability to chew and swallow
B. kitchen safety
C. anger management
D. recognition of family members
B. kitchen safety

When providing caregiver training to the spouse of an individual diagnosed with early-stage dementia, the OT will MOST likely need to instruct the caregiver in strategies to compensate for what deficits?

A. short-term memory
B. fine and gross motor skills
C. social skills
D. dressing skills
A. short-term memory

To develop the MOST relevant goals for a student’s school-based occupational therapy program, what should the OT focus on?

A. teacher’s perspective about why the student struggles in the classroom
B. student’s ability to access and participate in the curriculum
C. family’s priorities for their child
D. areas of delay identified in the occupational therapy evaluation
B. student’s ability to access and participate in the curriculum

An OT practitioner is planning a meal preparation activity for an individual with cognitive deficits in the areas of attentional and organizational skills. What is the most appropriate activity to use FIRST in addressing sequencing skills?

A. setting the table
B. planning a meal
C. baking cookies following a recipe
D. preparing a shopping list
C. baking cookies following a recipe

An OT practitioner is working with an individual in a work program setting. What is the FIRST step to achieving the program objective of preventing reinjury?

A. performing a prework screening
B. learning proper body mechanics
C. participating in work hardening
D. engaging in vocational counseling
B. learning proper body mechanics

An individual was unable to achieve the following goal: ” The client will initiate two requests to other group members for sharing materials within a 1-week period.” What would be the BEST revised goal for this client?

A. initiate two requests to other group members for sharing group materials within a 2-week period
B. initiate one request to one other group member for sharing group materials within a 1-week period
C. greet the other group members at the start of each group session
D. greet the group leader at the start of each group session
B. initiate one request to one other group member for sharing group materials within a 1-week period

A child with autism spectrum disorder demonstrates inadequate playground skills. She plays alone, generally repeating the same play each day. Which approach will the OT MOST likely select to promote her skills?

A. OT joins group on playground, facilitating child’s play and including other children
B. OT works with child on the playground when other students are not using the playground
C. recommend the parent enroll the child in ballet lessons after school
D. ask the physical education teacher to include her in small groups during gross motor activity
A. OT joins group on playground, facilitating child’s play and including other children

A child with learning disabilities resulting in low frustration tolerance and poor self-esteem is learning how to tie shoelaces. Which method is the MOST appropriate for the OT to introduce to this child?

A. physical guidance
B. verbal cues
C. backward chaining
D. forward chaining
C. backward chaining

When an individual with neurological deficits sits down to read a magazine in the living room of her group home, she becomes distracted by the conversations of her housemates. Which of the following activities BEST addresses the underlying cognitive problem?

A. playing a simple, repetitive card game in a quiet environment
B. measuring ingredients for a recipe while there is music playing
C. referring to a catalog and filling out a catalog form
D. walking and bouncing a ball simultaneously
B. measuring ingredients for a recipe while there is music playing

An OT is planning intervention for a student who has difficulty manipulating her pencil to erase errors on her papers. What activities should be included to improve her in-hand manipulation?

A. finger-to-palm translation
B. plan-to-finger translation
C. simple rotation
D. complex rotation
D. complex rotation

An OT practitioner is planing a vocational intervention program to assist an individual in a community mental health day program develop skills needed for obtaining employment. Which of the following would be the MOST relevant intervention to include?

A. self-assessment of work habits and personality characteristics
B. activities focused on time and stress management skills as well as practice of job-seeking strategies
C. educating the work supervisors about the individual’s needs and offering environmental modifications to maximize performance
D. expressive activities such as making a collage with pictures of different types of jobs
B. activities focused on time and stress management skills as well as practice of job-seeking strategies

An individual with Guillian-Barre acute syndrome was recently admitted to a rehabilitation unit and is expected to remain for 3 to 4 weeks. At what point in the rehabilitative process should the OT order adaptive equipment for this individual?

A. after the patient and family have accepted the individual’s disability
B. as soon as the insurance provider approves it
C. within the first week of therapy
D. just before discharge
D. just before discharge

An individual diagnosed with cancer has developed chemotherapy-induced neuropathy, resulting in wristdrop and decreased sensation in the RUE. The individual is right-handed. Which treatment approach is MOST appropriate for optimizing UE functional performance?

A. meaningful activities to improve right-hand pinch and grasp
B. RUE sensory reeducation
C. use of orthosis to provide support until wrist strength returns
D. change of dominance training
C. use of orthosis to provide support until wrist strength returns

An OT is planning a group program in an acute care psychiatry setting for severely mentally ill individuals who display disorganized thinking and difficulty functioning in many areas. What is the MOST appropriate type of group to use with these patients?

A. activity
B. psychoeducational
C. neurodevelopmental
D. directive
D. directive

An OT is concerned about a child’s inability to control flexion and extension of the arm when reaching for toys. He flexes or extends the arm too much, making accurate placement of the hand very difficult. When developing goals, what area is MOST likely in need of development?

A. ability to isolate movement
B. ability to grade movement
C. ability to control how much fast movement occurs
D. bilateral integration of arm movement
B. ability to grade movement

While evaluating an individual with arthitis, the OT observes PIP joint hyperextension and DIP joint flexion in the digits. The OT will MOST LIKELY document this as a:

A. boutonniere deformity
B. mallet finger deformity
C. congenital deformity
D. swan neck deformity
D. swan neck deformity

An OT is helping the parent of a 4-year-old with autism spectrum disorder to identify strategies to support grooming and hygiene. They are focused on ways to overcome difficulties during toothbrushing routines. What should the OT recommend?

A. advise that many children with autism spectrum disorder outgrow difficulties with toothbrushing
B. identify and trial several different strategies, such as various toothpaste flavors, use of electric toothbrush, pictures to show steps
C. give the child a choice between brushing his teeth and another, less-preferred activity
D. vary the schedule so the child doesn’t begin to associate negative activities with bedtime
B. identify and trial several different strategies, such as various toothpaste flavors, use of electric toothbrush, pictures to show steps

The goal for an individual in the later stages of Parkinson’s disease is to dress independently. The BEST adaptation to compensate for this person’s physical deficits would be:

A. hook and loop closures on front-opening clothing
B. large buttons on front-opening clothing
C. larger clothing slipped on over the head with no fasteners
D. stretchy fabric clothing with tie closures in the back
A. hook and loop closures on front-opening clothing

A patient with poor visual acuity is about to be discharged after completing a rehabilitation program following a total hip replacement. The MOST appropriate environmental adaptation to ensure that the individual can go up and down stairs safely is:

A. installing a stair glide
B. mounting handrails on both sides of the steps
C. marking the end of each step with high-contrast tape
D. instructing the patient to take only one step at a time when going up or down
C. marking the end of each step with high-contrast tape

A 7-year-old child with limited pincer grasp wants to zip his own pants in school because he gets embarrassed when he has to ask his teacher for assistance after using the bathroom. Which should the OT recommend the child try FIRST?

A. large key ring
B. oversized fasteners
C. colored zippers
D. hook and loop fasteners
A. large key ring

An elderly individual who was hospitalized for a right CVA with left upper extremity flacidity and decreased sensation is beginning to experience sensory return in the left upper extremity. What intervention strategies should now be included in the treatment plan?

A. remedial treatment, such as rubbing or stroking the involved extremity
B. remedial treatment, such as the use of hot mitts to avoid burns
C. compensatory treatment, such as testing bathwater with the uninvolved extremity
D. compensatory treatment, such as using a one-handed cutting board to avoid cutting the insenate hand.
A. remedial treatment, such as rubbing or stroking the involved extremity

A client arrives to exercise group wearing traditional garments that cover the body from head to toe, restricting movement while exercising. The OT observes the individual’s discomfort and is concerned about participation, but also wants the client to be comfortable in an environment that is obviously culturally different for her. What is the BEST action for the OT to take, demonstrating cultural competence in this situation?

A. when the opportunity arises, discreetly explain to her that dressing more appropriately for the group will allow for better participation
B. comment on the beauty of the clothes, asking for affirmation from the rest of the group
C. gently suggest that next time the client wear loose-fitting clothing that will be comfortable to exercise in
D. ask the individual if there are cultural guidelines for how to dress for exercise group
D. ask the individual if there are cultural guidelines for how to dress for exercise group

An OT is running a group for women with eating disorders. The activity is to create a gift box with a message inside about what they appreciate about another individual in the group. After accidentally tearing the side of her gift box, one client looks very distressed and stop participating. Which response is BEST for the OT to enable the client to continue working on her goal of self-acceptance?

A. set up a time to work with her one-on-one
B. take the box, make the necessary repairs, and encourage her to continue writing her message
C. discuss the symbolism of how we all experience breaking and healing
D. bring her the supplies to start over again, and leave out the message part of the activity
C. discuss the symbolism of how we all experience breaking and healing

An OT has been asked to design a health promotion group for individuals with cognitive impairment functioning as a parallel task group level. What is the MOST appropriate activity for the OT to use?

A. planning a dinner party
B. a “healthy eating” board game
C. a Weight Watchers support group
D. a therapist-led aerobics class
D. a therapist-led aerobics class

The OT is treating a child an above-elbow amputation who is experiencing hyper-sensitivity of the residual limb. The therapist would MOST likely perform which intervention in the preprosthetic phase of treatment?

A. engage the child in play activities that strengthen bilateral upper extremities
B. include activities to increase the range of motion in the should on the affected side
C. encourage play activities that incorporate taping, application of textures, and weight-bearing to the residual limb
D. practice dressing activities that include putting on and taking off the UE prosthesis
C. encourage play activities that incorporate taping, application of textures, and weight-bearing to the residual limb

An OT is assessing an individual with dysphagia. Which of the following should the OT address FIRST?

A. jaw pain and tooth-grinding habits
B. cranial nerve function assessment
C. mental status, oral structures, and motor control of head
D. muscle length control via finger-to-nose tests
C. mental status, oral structures, and motor control of head

A flight attendant with low back injury is participating in a work-hardening program. The individual can successfully simulate distributing magazines to all passengers in a plane using proper body mechanics. To upgrade the program gradually, what should the OT NEXT request that the individual simulate?

A. serving from a beverage cart
B. issuing blankets and pillows
C. distributing magazines to half of the passengers
D. putting luggage in a overhead compartment
A. serving from a beverage cart

An OT is fabricating an orthotic for an individual who presents with a low ulnar nerve injury lesion. The orthosis MOST appropriate for this individual is one that includes which component?

A. prevents hyperextension of the PIP joints and allows PIP flexion
B. prevents hyperextension of the MCP joints and allows MCP flexion
C. allows hyperextension of the MCP joints and prevents MCP flexion
D. allows flexion and hyperextension of the MCP joints
B. prevents hyperextension of the MCP joints and allows MCP flexion

An OT is seeing an individual with Stage IV cancer for palliative care. What will the emphasis of intervention be for this individual?

A. emphasizing quality of life and engagement in meaningful activity
B. improving strength and endurance and managing pain
C. maximizing independence in perineal activities
D. dealing with the psychological issues associated with preparing for death
A. emphasizing quality of life and engagement in meaningful activity

An elderly individual who ambulates with a walker in the home states he does not like sponge baths and would prefer to resume taking showers but is afraid of falling. Which of the following should the therapist do FIRST?

A.Suggest bathtub bathing instead of showering
B. encourage the client to purchase a shower chair
C. demonstrate how using a shower chair improves safety
D. explain that therapy will boost his confidence level when showering
C. demonstrate how using a shower chair improves safety

An OT is treating a restaurant worker with pain resulting from a cumulative trauma disorder. The OT suggested using elastic taping to decrease pain in the UE while working. In addition to decreasing pain, this type of taping can also assist with:

A. decreasing inflammation and edema
B. potentially increasing muscle fatigue
C. limiting ROM and strength
D. desensitizing the UE
A. decreasing inflammation and edema

An OT has provided a cup with a cutout area at the rim to a 6-year-old child with dysphagia. What is the BEST way to explain the purpose of the cutout in the cup to the family?

A. slow the drinking process
B. allow the chin to remain tucked when drinking
C. allow the caregiver to control the flow of liquid
D. minimize biting reflexes when the cup is placed in the mouth
B. allow the chin to remain tucked when drinking

A 6-year-old received OT for dressing skill development and is now independent. At discharge, what is the BEST advice for the OT to give the child’s parents to maintain the child’s independence in dressing at home?

A. give assistance when the child asks for it to provide a successful experience
B. provide praise for completed dressing; do not help the child get dressed
C. supply oversized clothing with hook and loop closures and large snaps
D. use verbal prompts when needed and help with closures only
B. provide praise for completed dressing; do not help the child get dressed

Which leisure activity BEST suit a sixth-grade student with juvenile rheumatoid arthritis to help him maintain range of motion?

A. swimming
B. basketball
C. soccer
D. aerobics
A. swimming

The OT has determined that progressive muscle relaxation training would benefit an individual who has anxiety and a limited attention span. What is the FIRST step in the training program?

A. make a fist, and then gradually relax it
B. focus on a rhythmic, repetitive word
C. walk until an increased heart rate is achieved
D. deeply inhale and slowly exhale
A. make a fist, and then gradually relax it

An OT is working with individual with amyotrophic later sclerosis who developed a sacral decubiti and has recently become too weak to turn himself in bed. What should the OT plan to do NEXT in regard to client/caregiver instruction?

A. begin a strengthening program
B. suggest that client and caregiver begin a wheelchair education program
C. teach the caregiver how to position the client safely
D.provide an environmental control unit to the client
C. teach the caregiver how to position the client safely

A hospital-based OT is working on discharge plans for a 2-year-old child with paraplegic spina bifida who has just started using a power wheelchair. Which community resource recommendation is the MOST critical for this child?

A. social service agency
B. wheelchair equipment vendor
C. family physician
D. early intervention program
B. wheelchair equipment vendor

A client presents with chronic, poststroke upper extremity edema of the right arm and hand. Which of the following would the OT most likely suggest to manage the edema?

A. manual edema mobilization
B. hot pack applications
C. paraffin treatments
D. sensory reeducation
A. manual edema mobilization

An individual with left upper extremity flaccidity is observed sitting at a table in his wheelchair at lunchtime with his left arm dangling over the side. The FIRST positioning strategy the OT should introduce to the individual is:

A. positioning the UE on the tabletop surface with Dycem
B. repositioning the arm back on the wheelchair armrest
C. using an arm sling
D. attaching a lap tray
A. positioning the UE on the tabletop surface with Dycem

An OT is discharging a 4-year-old child with cerebral palsy from a rehabilitation setting to home. What are the MOST appropriate instructions for the OT to provide to the family for maintaining correct jaw control while feeding the child from the side?

A. jaw opening and closing are controlled with your index and middle fingers; place your thumb on the child’s cheek
B. jaw opening and closing are controlled with your index and middle fingers; place your thumb on the child’s larynx for stability
C. jaw opening and closing are controlled with the palm of your hand on the child’s jaw cupping it gently
D. jaw opening and closing are controlled with your index and middle fingers; place your thumb on the child’s ear for stability
A. jaw opening and closing are controlled with your index and middle fingers; place your thumb on the child’s cheek

Individuals in a mental health clubhouse program are participating in a social skills training group focused on learning skills for meeting someone for the first time. The participants have completed the step in which they performed the role-play as it most recently occurred, without trying to use the most effective social skills. What is the NEXT action the OT should take when using a responsive social skills training protocol?

A. give a homework assignment focusing on meeting someone for the first time
B. provide positive and constructive feedback about performance in the role-play
C. discuss how to carry out strategies when meeting someone for the first time
D. runt he role-play again, this time using effective social skills for the same situation
B. provide positive and constructive feedback about performance in the role-play

The BEST way for an individual with hemiparesis and mild perceptual deficits to button a shirt is to:

A. button all the buttons before putting the shirt on
B. get the shirt all the way on, then line up the buttons and holes, and begin buttoning from the top
C. get the shirt all the way on, then line up the buttons and holes, and begin buttoning from the bottom
D. use a buttonhook with a built-up handle
C. get the shirt all the way on, then line up the buttons and holes, and begin buttoning from the bottom

An OT practitioner wants to provide functional activities as part of an individual’s hand rehabilitation program. Which of the following activities are appropriate?

A. active and self-care range-of-motion techniques
B. crafts, games, and self-care tasks
C. cone stacking, pegs, and pulleys
D. mild, moderate, and resistive Thera-Band exercises
B. crafts, games, and self-care tasks

The OT is observing a worker an intellectual disability perform a packaging task requiring assembly of a game box by placing first a pad of paper, then a pencil, then a plastic game piece into a box. The OT realized the client is having difficulty utilizing the correct assembly sequence. The OT decides that backward chaining would be the most effective technique for training this worker. How can the OT BEST introduce this technique?

A. instructing the worker to reverse the packaging sequence, placing the plastic game piece in first
B. prompting the worker to use the correct sequence with each item, then gradually eliminating prompts beginning with elimination of the prompt for the plastic piece
C. having the worker master the first step, putting the pad into the game package, then passing the package on to another worker to insert the pencil and the plastic piece
D. demonstrating and repeating the correct sequence before each of the worker’s attempts to package all three items
B. prompting the worker to use the correct sequence with each item, then gradually eliminating prompts beginning with elimination of the prompt for the plastic piece

The OT is working with the parents of a 4-year-old boy who demonstrates a strong tonic bite reflex when eating. What type of utensils will the OT MOST likely recommend to the child’s parents?

A. weighted universal grips
B. curved utensils
C. swivel utensils
D. rubber-coated spoons
D. rubber-coated spoons

Participants from a partial hospitalization program are taking a outing into the community. One program member complains of shortness of breath. The OT practitioner determines the individual’s resting heart rate is 128 bpm, and blood pressure is 230/180 mm Hg. What is the MOST appropriate action for the OT to take?

A. continue the community re-entry outing watching to see if additional symptoms emerge
B. immediately return the rest of the clients to the day program
C. help the patient lie down and wait until his vital signs return to normal
D. activate the emergency response by calling 911
D. activate the emergency response by calling 911

Which of the following activities would BEST represent an expected outcome for an individual who completes an energy-conservation program?

A. getting dressed without becoming fatigued
B. lifting heavy cookware without pain
C. doing handicrafts without damaging his or her joints
D. dusting and vacuuming more quickly
A. getting dressed without becoming fatigued

An OT is working with an individual who is s/p RUE shoulder replacement. The client is right-hand dominant and an avid woodworker. Currently, limited RUE ROM prevents participation in woodworking, which requires at least 120 degrees of shoulder flexion. The OT and the patient collaboratively develop a plan that incorporates woodworking to increase RUE ROM. After establishing the “just right challenge” at 40 degree of shoulder flexion for sanding the pieces of a wood project, how should the OT progress the patient in order to achieve his goal?

A. progress him to the use of power tools
B. apply increasingly heavier wrist weights during sanding and staining activities
C. provide him with tasks that require bilateral coordination
D. involve him in sanding and staining increasingly larger pieces
D. involve him in sanding and staining increasingly larger pieces

The OT has been working with a group of individuals with co-occurring mental illness and substance abuse in a clubhouse model for several months. The group is functioning at a mature level. Which is the BEST way for the OT to be involved in this group?

A. functioning as a peer in the group
B. functioning as a group leader
C. function as group advisor
D. functioning as group facilitator
A. functioning as a peer in the group

A man with arthritis in his hands i attending outpatient OT to learn joint protection techniques. He wants to continue his hobby of needlepoint, which he does every day during his train commute to and from work. What advice should the OT give him regarding his hobby?

A. provide him with needlepoint designs that have a low level of complexity
B. teach him to take breaks frequently when doing needlepoint and to respect pain
C. encourage him to take up a different hobby that does not require him to hold small items, such as a needle
D. recommend he do only small needlepoint projects that can be completed in less than an hour.
B. teach him to take breaks frequently when doing needlepoint and to respect pain

A child with developmental delay in a kindergarten class has just developed the strength and stability in high right hand to hold the scissors properly and make snips in paper. Which activity would help the child to develop the next level of scissor skills?

A. cut cloth and cardboard
B. cut along curved lines
C. cut along straight lines to cut out a triangle
D. cut the paper in two following a straight line
D. cut the paper in two following a straight line

The OT fitted a 6-year-old child with an adapted seat to use during mealtime and other tabletop activities at home. What information is MOST appropriate to convey to the parents?

A. use the seat as the child is willing
B. bring the seat in for each weekly therapy session
C. bring the seat in for reevaluation in 4 to 6 months
D. Keep the seat until the end of the IEP
C. bring the seat in for reevaluation in 4 to 6 months

An individual preparing for discharge following a brief inpatient hospitalization for depression describes to the OT the type of services she would like to be involved in postdischarge. She is interested in structuring her day around work but does not feel she is ready for paid employment. She enjoys being in the company of others and does not feel she will need the support of professional mental health providers. Which setting best meets her criteria?

A. transitional employment
B. clubhouse model
C.. partial hospitalization
D. group home
B. clubhouse model

During a task group, an individual diagnosed with borderline personality disorder tells one group member that their project “looks like it was made by a little kid” and tells another group member that their project “looks stupid” and tosses the project across the room. Which is the BEST approach for the OT to take with the person who is displaying this behavior?

A. ban the individual from group activities until her behavior improves
B. tell the individual how disappointed you are in her behavior
C. work with the individual on appropriate communication skills
D. ask the individual how she thinks the other group members felt when she did that
C. work with the individual on appropriate communication skills

When instructing the parents of a toddler in the use and care of a hand orthosis, on which instructions should the OT place the most emphasis?

A. checking for irritation and pressure problems
B. avoiding excessive heat exposure
C. cleansing the orthosis regularly
D. adhering strictly to the wearing schedule
A. checking for irritation and pressure problems

A 7-year-old child with cerebral palsy demonstrates fair sitting stability and good head control with fluctuating lower extremity extensor tone. What position device would the OT MOST likely use during feeding?

A. prone stander with lateral trunk supports
B. Rifton child’s chair with footrest and padded abductor post
C. caregiver’s lap, with one arm stabilizing his trunk
D. beanbag chair for full sitting support
B. Rifton child’s chair with footrest and padded abductor post

An OT is preparing to do a parachute activity as part of a sensory integration program. Because several of the patients in the group are taking antipsychotic medications, the OT should be alert for which possible side effect that could occur as a result of this activity?

A. orthostatic hypotension
B. photosensitivity
C. excessive thirst
D. blurred vision
A. orthostatic hypotension

A very confused long-term care facility resident is frequently found in the rooms of other residents in the middle of the night. Which of the following environmental adaptations would MOST effectively prevent wandering?

A. apply wrist restraints after the client has fallen asleep
B. keep hallways clear of obstructions to prevent injury
C. install an alarm on the client’s door
D. move the individual’s room close to the nurses station
C. install an alarm on the client’s door

An OT has been hired by a community residence for women with mental health issues as a consultant to address the problem of low motivation and low activity levels in the areas of activities of daily living and instrumental activities of daily living and instrumental activities of daily living. What approach is MOST appropriate?

A. provide occupational therapy treatment to increase occupational performance in the areas identified
B. develop a plan with staff to change the social environment to one that will enhance motivation and activity levels
C. design a range of living skills groups so that every resident will be included
D. help the residents to achieve personal goals, make decisions, or change behaviors
B. develop a plan with staff to change the social environment to one that will enhance motivation and activity levels

An OT is working with a 6-year-old child who occasionally drops his utensils when eating because of a slight decrease in hand range of motion/grasp limitations. Which piece of equipment would the OT MOST likely recommend FIRST?

A. swivel utensil
B. pediatric universal holders
C. foam tubing around the utensils
D. weighted utensils
C. foam tubing around the utensils

During an infant’s OT session, the mother reports she has observed that her baby has difficulty with swallowing and frequently chokes. To reduce the risk of aspiration and facilitate swallowing the OT should keep the head in what position?

A. neutral position
B. slightly flexed
C. slightly extended
D. rotated toward the feeder
B. slightly flexed

A 60-year-old auto mechanic with diabetes and impaired sensation in the residual lower limb has been referred to OT for prosthetic training following an above-knee amputation. The FIRST item the OT practitioner should address is:

A. skin inspection
B. grooming techniques
C. retirement planning
D. returning to work
A. skin inspection

the OT and client have agreed on a baking activity to work on decision-making skills, but when the time comes. the individual is reluctant to participate. Which of the following approaches would be MOST appropriate?

A. premeasure all the dry and liquid ingredients
B. use a recipe with no more than 3 steps
C. offer a choice of slice-and-bake cookies or a cake mix
D. create a checklist to use as each ingredient is added
C. offer a choice of slice-and-bake cookies or a cake mix

A client who presents with little to no active range of motion in the left shoulder after a CVA requires the OT to perform passive range of motion. How wold the OT proceed?

A. teaching the client how to utilize a wall-mounted shoulder wheel
B. having the OT move the shoulder joint through its full range of motion
C. using an overhead pulley system
D. training with an arm ergometer
B. having the OT move the shoulder joint through its full range of motion

An individual with bipolar disorder demonstrates frequent verbal outbursts and has been referred to occupational therapy to work on developing social skills. When using a Dialectical Behavior Therapy approach, what is the MOST appropriate forms to use?

A. individual treatment sessions in a quiet place such as the individual’s room
B. a group format following a specified protocol
C. individual treatment sessions that allow for flexibility within each session
D. a group format that alternates leadership opportunities between group members
B. a group format following a specified protocol

An OT is applying PNF techniques for weight shifting during an activity that requires an individual to use the right hand to remove groceries from a bag on the floor to the right. The MOST benefit would be gained from this activity by then placing the groceries:

A. on the counter directly in front
B. on the counter to the left side
C. in the upper cabinet to the right side
D. in the upper cabinet to the left side

A student is learning to activate a switch for a communications device. Although the switch is mounted on the wheelchair tray, the student continues to have difficulty operating it because of excessive muscle tone. Despite practicing for extended periods of time, the student is not making any progress. What should the OT do next?

A. reposition the switch to facilitate easy access and adjust further as needed
B. passively stretch the student’s upper extremity to increase range of motion
C. use a brightly colored switch to increase visibility
D. use systematic behavioral reinforcement through shaping

A 5 year old child who has mild developmental delay is learning to self-dress. Most recently, the child has mastered putting on and taking off a coat, as well as buttoning and unbuttoning it. From the dressing tasks listed, which should the child learn to do next based on the typical development sequence for dressing?
Pulling up a jacket zipper would be the next dressing skill the child should learn.

An entry level OTR who works in a long-term care facility is screening a resident who has ALS and uses a wheelchair. The resident’s UE functional strength is Trace (1/5) on the left and Poor Minus (2-/5) on the right. The resident wants to eat meals in the facility dining room but is embarrassed about frequently dropping assistive eating devices and having several incidences of coughing while eating. What care coordination task should the OTR complete FIRST?
Coordinate with the multi-disciplinary team to identify specific feeding and swallowing objectives.

An entry level OTR who works in a long-term care facility is screening a resident who has ALS and uses a wheelchair. The resident’s UE functional strength is Trace (1/5) on the left and Poor Minus (2-/5) on the right. The resident wants to eat meals in the facility dining room but is embarrassed about frequently dropping assistive eating devices and having several incidences of coughing while eating. What symptom is the resident most at risk for experiencing during a meal?
Variation in voice quality or loss of voice after swallowing, which is a symptom of dysphagia.

Which factor is MOST IMPORTANT to determine when considering if the resident would benefit from a mobile arm support?
Amount of upper extremity passive joint mobility

At a minimum, what information should the OTR document to increase the likelihood of additional authorized visits?
Functional problem statements indicating the need for skilled services

How should the OTR respond to the client’s concerns about splinting?
Encourage full participation in daily tasks by collaborating on ways to modify typical daily activities

Your patient had a CVA several weeks ago. Which task oriented approach would be most beneficial to include as part of the intervention plan for maximizing the client’s physical abilities?
Constraint-induced movement therapy is a task-oriented approach to motor acquisition.

For a patient who had a CVA several weeks ago, which type of activity should be included as part of the INITIAL intervention to REMEDIATE the client’s motor impairment?
Weight bearing through the affected upper extremity prior to an activity

Which assessment should be included as part of the initial evaluation and can be administered by the COTA?
COPM

What is contraindicated during phase I of cardiac rehab?
Isometric exercises can cause a rapid and sudden increase in blood pressure

AOTA
Publishes Standards of Practice for Occupational Therapy and the Reference Manual of official Documents of the AOTA.

NBCOT
Responsible for issues related to certification and certification renewal

ACOTE
responsible for accreditation of OT education standards in support of the practice standards set forth by the AOTA

Negligible Correlation
0 – .2

low correlation
.2 – .4

moderate correlation
.4 – .6

high correlation
.6 – .8

.8 – 1.0
very high correlation

Constructional Apraxia
have difficulty constructing/assembling objects

A client has Poor minus (2-/5) functional strength of the dominant UE. What is the most effective method for grading an activity to improve the client’s muscular strength?
Increasing the amount of time an isometric contraction is held

What is the PRIMARY purpose for completing a dysphagia screening for an inpatient who had a right CVA several days ago?
To observe the patient’s oral motor control and determine a need for further evaluation. A screening is not used for diagnostics or treatment planning – it allows the OTR to quickly detect the presence of a deficit and determine if an in-depth evaluation is needed.

An OTR is preparing to interview an outpatient who has a TBI and is functioning at Rancho Level VII. What charcteristic of an individual’s cognitive behavioral function is typically included in the description of this level?
Consistent orientation to time and space. Patients at this level frequently display robot-like behaviors and require minimal assistance for routine ADL.

Cognitive Behavioral Therapy
The individual can change their behavior by first challenging their negative assumptions (cognitions) relating to trigger events.

If a client presents with a boutonniere type deformity, what is the most likely cause?
Rupture or lengthening of the central slip of the EDC

Rupture of the flexor digitorum superficialis tendon or lengthening of the lateral slips of the EDC may result in…
swan neck deformity

An OTR who receives a referral for a 5 yr old with PDD should first gather background information on the child by:
discussing the child’s history with the caregiver to identify the child’s present level of performance in different occupational roles

When considering the readiness of a client for a power wheelchair, the OTR should primarily be concerned with
Cognitive ability, to ensure that the individual has the awareness, problem solving, and decision making capabilities to learn how to consistently and safely operate the chair

agoraphobia
a morbid fear of open spaces (as fear of being caught alone in some public place)

During an OT session at an outpatient facility, the client states that being injured has caused considerable stress at home. The client often feels frustrated and angry and reports a concern for having to physically discipline her children. The FIRST action that the OTR should take is to:
further explore the situation since the child may be at risk. Gathering specific information about the client’s comment is critical due to the seriousness of the accusation. Therapists are required by law to report any situation where abuse is suspected. Documentation occurs after all information is obtained from the individual. More information is needed before any action is taken.

According to SI frame of reference, tx is most effective when:
a child directs his or her actions within an environment structured by an OTR that provides the just-right challenge. The child’s skills and abilities are facilitated in an environment where the child is directing their participation in activities

An individual who is referred to OT to increase socialization and develop leisure skills sits alone, despite the active involvement of the other group members. To increase the individual’s involvement in the group, the OTR’s MOST APPROPRIATE FIRST action would be to
review the goals and purpose of the group with the individual. In order for the individual to become involved in a group process, the group norms and expectations must first be explained to the client.

The BEST position in which to feed an 8-month old with severe hypotonia is
semireclined with the neck in neutral and head at midline. This will promote correct alignment of body parts during feeding and requires minimal active muscular control

An individual has difficulty conceptualizing and organizing future events, and is experiencing decreased feelings of competence. The BEST treatment intervention is to have the individual:
prioritize short term goals with a high chance of success. Feelings of competence are enhanced by successful progression toward achievable goals

A 15 month old child with motor delays is able to get into a sitting postition, but does not pull up to standing. Which developemental milestone is the OTR MOST LIKELY to emphasize next?
Creeping – The Developmental Frame of Reference suggests that once a child has achieved the developmental skill of sitting, the next skill to achieve is creeping.

Before recommending purchase of adapted switches for a 3 year old with severe cerebral palsy, the OTR must FIRST:
determine goals – considering how a new piece of equipment will fit into a person’s occupational roles should be the first consideration for identifying equipment needs

When positioning an individual with flaccid hemiparesis in a wheelchair, an OTR using NDT would place the affected arm in:
a supported position, with the scapula protracted and the arm forward to inhibit abnormal patterns of movement and inhibit synergistic posturing and help to maintain scapular mobility for normal movement patterns

An OTR is working on pencil grasp with an 8 yr old. The child consistently uses a palmer grasp. The MOST EFFECTIVE activity to facilitate the development of a more mature pencil grasp pattern is:
stringing small beads would be most effective way to improve functional coordination and use of intrinsic muscles of the hand. This activity would also promote a more refined distal fingertip control. Distal control in intrinsic muscle strength and coordination are important components of a mature pencil grasp.

When learning one-handed dressing techniques, an individual with right hemiplegia, aphasia, and ideational apraxia would MOST LIKELY benefit from which of the following treatment approaches?
Tactile cueing for initiating dress because ti uses the individual’s existing strengths to minimize the functional deficits resulting from aphasia and ideational apraxia

To maximize independence in feeding, an individual with a complete C-5 quadriplegia will most likely benefit from a:
plate guard, a wrist-hand orthosis with a utensil slot, bent angled utensils, and a long straw with a straw holder. A wrist-hand orthosis would compensate for the absence of wrist extensors/flexors. Bent angled utensils aid in maximizing self-feeding skills by compensating for the absence of radial and ulnar deviation. A plate guard provides a barrier against which the individual could push food to get the food onto a fork.

C5
will have the following movements: elbow flexion, forearm supination, shoulder external rotation, should abduction to 90 degrees.

What technique helps to normalize tone and promote sensory feedback in a pt who had a CVA with resulting hypertonicity?
weight bearing/joint compression is an inhibitory technique described in both Rood’s and Bobath’s theories

The parents of a 6 month old who has mild spasticity have been instructed in a positioning program for home. The BEST way to assess the effectiveness of the intervention program is to:
have the parents demonstrate correct positioning

Based on a child’s performance, a school-based OTR believes that program modifications in the IEP are needed. The first course of action the OT should take should be to:
Prepare a summary of the additional needs for the multidisciplinary team

Based upon the initial evaluation results of a patient with a brain tumor, independence in upper body dressing is established as a short-term goal. After 2 weeks of treatment the patient is not progressing as quickly as expected. The FIRST thing the OTR should do is:
contact the physician to discuss the patient’s progress. It may be appropriate to modify the patient’s plan but the OTR should FIRST discuss this with the physician

An OTR emplyed by a multi-state entertainment theme park corporation can FIRST enhance the corporation’s awareness of their role in assuring parks accessibility and ADA compliance by:
identifying barriers which inhibit customer use. Determine occupational needs of the organization. The first step of the process would be to identify the needs and issues surrounding parks accessibility and ADA compliance.

An OTR working in a long-term care facility is asked to recommend seating approaches for the residents in the dining program. The BEST approach to meet the needs of the entire group would be to:
assess the resident’s needs and strengths. Determine occupational needs of the program.

When does Asymmetric Tonic Neck Reflex disappear?
4 months

When does the stepping reflex disappear?
2 months

When does palmar grasp reflex (spontaneous grasp of adult fingers) disappear?
3-4 months

When can a child lift self by arms when prone?
2 months

When can child sit alone?
6-7 months

When can child crawl?
7 months

When can child pull to stand?
8 months

WHen can child stand alone?
11 months

When can child build a tower of 2 cubes?
13 months

Reaching at 3 months
Voluntary reaching gradually improves in accuracy. It does not require visual guidance of arms and hands, but rather a sense of movement and location.

Reaching at 9 months
Infant can redirect reaching to obtain a moving object that changes direction.

When can a baby roll from tummy to back and back to tummy?
4-5 months

What is the primary purpose of the chin tuck?
To prevent food secretion from entering the larynx below the level of the vocal chords

A client who has motor deficits secondary to the onset of
Huntington’s disease is participating in OT. One of the client’s
goals is to be independent with dressing. Which intervention
activity is BEST to include as part of the initial intervention
plan?
Educate the client and caregivers about methods for adapting the fasteners on the client’s clothing. HD is a progressive neurodegenerative disorder. Symptoms
include choreo-athetoid movements and alterations in behavior and cognitive functions. A client in the early stages would benefit most from learning techniques to compensate for motor deficits.

A 5-year-old child who has mild developmental delay is learning to self-dress. Most recently, the child has mastered putting on and taking off a coat, as well as buttoning and unbuttoning it. From the dressing tasks
listed, which should a child learn to do NEXT based on the typical developmental sequence for dressing?
Pulling up a jacket zipper

Minimum width of a hall for a wheelchair
36 in

Minimum clear width for a wheelchair is _ inches for a door.
32

If wheel chair ramp changes direction the minimum size of the landing is:
5′ by 5′

Wheel chair hand rail height
33″ to 36″

Standard wheelchair width
24-27″

Phase I Cardiac Rehab
Acute care
-3-5 day hospital stay for uncomplicated MI
-initiate activity once stable for 24hrs
-low intensity (2-3 METs) progressing to 3-5 METs by discharge
-lifting restricted for 6 wks
-monitored low level physical activity
-self-care
-reinforcement of cardiac & post-surgical precautions
-energy conservation education
-graded activity

Phase II Cardiac Rehab
Pt. is out of the hospital and goes to a rehab center. Target HR during rehab is 10bpm below the rate of ischemic change. Subacute (outpatient) rehab; most payers allow 36 visits (3x/wk for 12 wks); for pts at risk for arrhythmias or angina that benefit from ECG monitoring; 30-60 min session with warm up and cool down; gradually weaned to self-monitoring; exit point is 9 METS (5 METS for most daily activities); strength training after 3 wks of rehab, 5 wks post-MI, or 8wks post-CABG; elastic bands or 1-3 lb weights, to mod loads of 12-15 reps

Phase III Cardiac Rehab
Community exercise program, post phase II rehab; entry level criteria is 5 METs, clinically stable angina, and arrhythmias controlled with meds; progress to self-regulation; 50-85% capacity 3-4 times/wk for 45 min sessions; discharge in 6-12 mo; 5 wks post-surgical or MI; 3 wks post-PTCA; no evidence of unstable s/s; resistance with 50% or more of 1 RM, 1-5 lb weights, RPE 11-13; RPP prescribed by ETT

Fine Motor Development 8 month
mouths toys, accurately reaches for toys, transfers toys hand to hand, bangs objects together, grasps small objects in fingertips

Gross Motor Development 8 months
sits independently, rolls, stands with support, and crawls on belly

When does the Moro Reflex disappear?
3-4 months

symmetric tonic neck reflex
onset: 4-6 months, disappears: 8-12 months
Stimulus: place infant in the crawling position and extend the head
Response: Flexion of the hips and knees
*Breaks up total extensor posture

When should head lag stop when lifting the baby up to sitting from a lying down position?
6 months

equilibrium reactions
stimulus: displace center of gravity by tipping support surface, response: righting of head/trunk/limbs

When does the asymmetric tonic neck reflex appear and disappear?
present at 1 month, disappears at 4 months

asymmetric tonic neck reflex
Onset age: 37 wks gestation
Integration age: 4-6 months
Stimulus: Fully rotate infant’s head and hold for 5 seconds.
Response: Extension of extremities on the face side, flexion of extremities on the skull side.
Relevance: Promotes visual hand regard.

flexion righting reaction
when the child is pulled to a sitting position, the development of antigravity neck flexion is associated with the child’s ability to maintain head and trunk alignment against the pull of gravity

support reaction
the ability to co-activate muscle groups of the appropriate extremity to support body weight or posture in a certain position

dysesthesia
Touch sensation experienced as pain

backward chaining
A teaching procedure in which a trainer completes all but the last behavior in a chain, which is performed by the learner, who then receives reinforcement for completing the chain. When the learner shows competence in performing the final step in the chain, the trainer performs all but the last two behaviors in the chain, the learner emits the final two steps to complete the chain, and reinforcement is delivered. This sequence is continued until the learner completes the entire chain independently.

chorea
Sudden, jerky, purposeless movements. Characteristic of basal ganglia lesion (e.g., Huntington’s disease) (Chorea = dancing (Greek). Think choral dancing or choreography)

Huntington’s Disease
a human genetic disease caused by a dominant allele; characterized by uncontrollable body movements and degeneration of the nervous system; characterized with ataxic gait with choreoathetoid movements, rigidity develops with progression

ideational apraxia
regarded as higher level disorder in which the individual movements are intact but the execution of complex sequential movements with objects is impaired.
Considered severe from of ideomotor apraxia
misuse of objects can be seen in confusional states & dementia & may have little to do with apraxia

ideomotor apraxia
a condition where a person plans a movement or task, but cannot volitionally perform it. Automatic movement may occur, however, a person cannot impose additional movement on command

short form 36 health survey
-survey used to measure general health and well being
-it has been used in medical outcomes studies and is sensitive to change in health status
(i.e. to determine if participation in rehab is beneficial to a clients health, well being, and general quality of life)

While assessing the motor skills of an 8 month old child, the OT observes him assume a quadruped position and then begin to rock back and forth. This behavior most likely indicates:
normal development

while standing and holding onto furniture, a 3 yr old boy with delayed motor development shifts his weight onto one leg and steps to the side w the other. This movement is
cruising

During an initial interview parents describe their child as having severe difficulty in communicating and interacting with other and also exhibits repetitive and ritualistic behaviors. This is most likely
PDD

Can a child with a flaccid bladder be toilet trained?
No

MOHO primarily focuses on:
the effect of personal traits and the environment on role performance

An infant born 15 weeks premature has a hx of mult medical issues including retinopathy of prematurity, mechanical ventilation for 5 weeks, and poor feeding skills. The infant is now 43 weeks old, medically stable and engaging infant, with a g-tube and O2 supplement of 2 L by nasal cannula. What is the MOST appropriate intervention to pursue at this time?
Multisensory input, because an infant reaching full term or post term needs a multisensory diet to assist with age appropriate motor and behavioral skill acquisition and it is often found tht premature infants are limited in the amount of social interaction and sensory stimuli they receive since they have necessary medical equipment etc. If the infant were 32 to 35 weeks they would need range of motion and positioning, because of immature sensory systems and compromised respiratory systems.

An individual with strong dependency needs is able to lace a leather wallet only with consistent verbal cueing. Which is the BEST way to grade the activity in order to decrease dependency?
Ask the individual to try some lacing with distant supervision and praise her for what she has been able to do. The next step would then be to tell the individual to complete some lacing while the OT assists another pt in the same room.

When working with a child who is at risk for shunt malfunction, it is MOST important for the therapist to observe for:
unexplained sensory loss

For self-feeding with C5 spinal cord injury, what piece of equipment would be MOST appropriate?
Mobile Arm Support

When planning intervention for individuals int he acute phase of cardiac rehab, it is most important for the OT to select activities that:
Decrease the effects of prolonged inactivity, such as thromboembolism, orthostatic hypotension, and muscle atrophy

What is the MOST important when using a remotivation approach with a group of elderly individuals?
Use of photos, music, and discussion to encourage discussion of memories

A child with ADHD also exhibits perceptual deficits. What activity would be MOST effective intervention to train visual attention?
Blowing cotton balls at a target because child with ADHD have difficulty with sustained attention and effort and blowing cottons balls at a target is a short-term activity with immediate reward for successful completion rather than delayed gratification

An OT is training an adult worker with a developmental disability to put a pencil in the box before putting in the score pad in the box for a game packing task in a sheltered workshop. The pt has not done this before. What type of reinforcement schedule would be BEST to achieve the goal oflearning this task sequence?
continuous reinforcement of correct responses – continuous reinforcement is best for learning new behaviors, intermittent reinforcement is best for maintaining behaviors.

A pt with Guillain Barre is expected to stay in hospital 3-4 weeks. When should the OT practitioner order adaptive equipment?
Just before discharge to accurately determine the pt’s needs

For a pt at ACL 4, what is the BEST method for introducing an assembly activity?
Provide project samples for clients to duplicate

For a pt at ACL 3, what is the BEST method for introducing an assembly activity?
Provide repetitive, one-step activities. Pt will not likely produce consistent results.

For a pt at ACL 5, what is the BEST method for introducing an assembly activity?
Demonstrate a 3 step assembly process

For a pt at ACL 6, what is the BEST method for introducing an assembly activity?
Provide written directions for the individuals to follow

A toddler has feeding difficulties because of deficient oral-motor control and oral defensiveness but the parents want him to be able to eat cut up meats and veggies. The OT tells the parents the child can start eating these foods when their child is able to tolerate what?
Dry cereals with milk.

How to increase oral tolerance and control of food
Gradually modify textures from smooth and consistent (strained fruits and veggies) to smooth and slightly varied (apple sauce and mashed bananas and scrambled eggs) to increasingly resistive foods and a combination of contrasts such as hard and crunchy mixed with soft or liquid (i.e. dry cereal and milk). After the child has mastered this level of control and tolerance, he can safely proceed to an even greater variety of textures, tastes, and temperatures offered at family meals.

After a total hip replacement how does the pt perform a passenger side car transfer?
Back up the body to the passenger seat, hold onto a stable section of the car, extend the involved leg, and slowly sit in the car.

What is the ONLY activity that will facilitate hand function in the preparation phase?
Weight-bearing on hands

What help improve the ability of someone with a C6 or C7 SCI to hold something?
Wrist splints to promote development of tenodesis by allowing the finger flexors to shorten so the person will have a functional grasp when the wrist is extended.

An OT is positioning a child with poor muscle tone and postural instability into a prone stander to develop head righting. The child rapidly shows fatigue. How can the therapist BEST adjust the stander to decrease these reactions while continuing to address the goal of head righting?
Position the stander at 75 to 90 degrees from the floor so that the child doesn’t have to work against gravity a lot, but isn’t standing up straight which would be the easiest.

What is an effective strategy for increasing attention to the left for a person who has a diagnosis of unilateral neglect?
Encouraging available hemiplegic limb movements before or during a task because any movement will activate additional motor units which will then increase attention to the Left

What is the best height for a toilet for someone with lower extremity paralysis in a standard wheelchair?
18 inches which is the height of a standard wheelchair seat, making transfers easier

ACMRDD
Accredation Council for services for the Mentally Retarded and other Developmentally Disabled Persons

JCAHO
Joint Commission of Accreditation of Hospital Organizations reviews medical care of hospitals, psychiatric facilities, hospices, long-term care agencies, and MR/DD programs seeking accreditation.

CARF
Commission on Accreditation of Rehab Facilities reviews programs in free standing facilities as well as those that are part of a hospital system

What is the maximum height a door threshold can be for a person in a wheelchair?
Half an inch, beveled

Program evaluation
a systematic collection and reporting of outcomes data to document program effectiveness and cost-efficiency

An OT is evaluating a pt who had a CVA. The sensory portion of the test would be invalid for an individual with:
Receptive aphasia, because pt w receptive aphasia cannot comprehend spoken or written language so they couldn’t understand verbal directions or consistently respond to stimuli

A preschool child with spastic cerebral palsy uses “bunny hopping” for functional mobility during an OT evaluation. This indicates that a primitive reflex pattern is being used for mobility. What reflex is most likely being used?
Symmetric Tonic Neck Reflex: Because it assists the child in a quadruped position. It provides bilateral arm extension and hip flexion with the head raised (and bilateral arm flexion and hip extension when the head is lowered), which can be used to move forward.

What performance components are MOST important to consider when analyzing activities for use with adults with psychosocial problems?
The amount of self-control demands, time management demands, self-expression opportunities, and interest in the activity.

Phases of Swallowing
1) oral preparatory
2) oral
3) pharyngeal
4) esophageal

A preschooler with a dx of developmental delay is very withdrawn and passive. While working on toilet skills, the child reaches out for a toothbrush and starts to brush her hair with it. The OT recognizes the primary importance of this behavior as:
indicating initiative and beginning task-directed behavior. Because the child is so withdrawn, any spontaneous action should be seen as a positive sign. It is very important to encourage the child in independent exploratory behavior in order to develop task competence and become less withdrawn.

Directive group treatment is MOST appropriate in acute care mental health for individuals with:
Disorganized psychosis: directive group tx is a highly structured approach that is used in acute care psychiatry for minimally functioning individuals. It is useful for disorganized and disturbed functioning with pts with psychoses.

What should pt on neuroleptic meds avoid?
Individuals experiencing extrapyramidal syndrome, which may cause muscular rigidity, tremors, and/or sudden muscle spasms, should avoid using power tools or sharp instruments. Also pts on neuroleptic meds will have increased sensitivity to the sun and should avoid too much sunlight.

What is an example of a motor response to pain?
repeated protecting of the joints while moving

An OT is administering a standardized test to a yong client who suddenly becomes uncooperative and complains that the test is “too hard.” The MOST appropriate response for the OT would be to:
follow administration instructions and note changes in behavior

What is a common side effect of anti-anxiety medication?
confusion

The OT tx that would most likely meet the needs experiences by individuals with substance abuse problems is to:
assist with skill development in the areas of leisure, cognition, and perception, self-expression, and ADL

Protective reaction when falling forward includes:
shoulder flexion, internal rotation, and shoulder adduction

The goal for an adolescent with anorexia is to improve self-concept. Which component of meal preparation activity BEST addresses this goal?
State strengths and limitations regarding performance in the activity. Self-concept is defined as the value of one’s physical and emotional self. Stating one’s strengths and limitations about their performance is a reflection of self-concept.

An older adult with diabetes is working on a craft project involving cords as a way of increasing standing tolerance. The MOST relevant safety factor for the OT to take into consideration is the:
texture of the cords because individuals with diabetes frequently have poor circulation and sensation in their extremities so skin damage should be avoided since healing is compromised.

An older adult with back pain is working on a craft project involving cords as a way of increasing standing tolerance. The MOST relevant safety factor for the OT to take into consideration is the:
type of surface the pt stands on

An older adult with limited range of motion is working on a craft project involving cords as a way of increasing standing tolerance. The MOST relevant safety factor for the OT to take into consideration is the:
length of cord

An older adult with limited hand function is working on a craft project involving cords as a way of increasing standing tolerance. The MOST relevant safety factor for the OT to take into consideration is the:
width of cord

When planning a therapeutic program for a child who has deficits in visual discrimination, the first step is to provide matching activities that require:
the ability to recognize objects, because a child must be able to recognize an object before discriminating among its specific visual attributes

An OT is using a visual perceptual frame of reference. What would be the first step in planning a program for a child with visual perceptual problems?
Visual attention skills because the prepare and provide foundation skills for other aspects of visual perception. Once visual attention develops, the child can develop visual memory then general and specific visual perceptual skills.

A resident in a long-term care facility has severe attention span deficits that impair the ability to participate in self-feeding. The OT is most likely to recommend what method?
hand-over-hand assistance

In a long-term care facility an elderly resident with dementia repeatedly asks for her mother and becomes increasingly upset. The MOST therapeutic strategy for responding to this resident is to:
respond to the emotional tone expressed by the words: provide extra attention and reassurance

A pt has been referred to OT after open heart surgery and a period of prolonged bedrest. After the pt is able to tolerate sitting at the edge of the bed unsupported, the NEXT activity to OT should introduce is:
walking at 1mph (based on MET levels)

To avoid overstimulation when handling a stable, 12 week premature infant in the NICU setting, an OT practitioner must FIRST:
provide gentle human touch to enable the infant to slowly respond to intervention because the tactile system is the first to develop

An individual with depression is ready to return to the job held before taking a leave of absence. What is the FIRST action the OT should take?
Perform a job analysis. The OT can then work with the pt to maximize performance or request reasonable accommodation.

Medical documentation should be:
concise, objective, and accurate

A COTA and OTR have worked together the past 5 years. What is the supervisory process at this level?
A mutual process

A client uses a wheelchair and is independent in transfers and basic ADL. With continued OT intervention he is expected to function independently in the community. The MOST appropriate community living option for him, at this time, would be a:
transitional living center

Transitional Living Center
provide temporary living arrangements for individuals in the transitional phase between hospital and independent community living

Cradle-to-Grave homes
houses designed with accessibility in mind at the time of construction. Should an individual decide to use a wheelchair later in life his home would already be accessible.

Adult Day Programs
rehab oriented day programs in the community. Not residential.

An OT is developing a department in a rural long-term care facility. When developing the policies and procedures for documentation in the facility, what document would be MOST useful?
Uniform Terminology for Occupational Therapy: a document that defines OT in relationship to performance areas and performance components. Provides a common language for OTs to use to describe individuals and their performance components.

Assessment of Motor and Process Skills
Patient chooses from 63 ADL tasks, OT observes 2-3 tasks, allows therapist to determine ability of client, while taking into account the relative challenge of each or the ADL tasks the client performed.

Barthel Index
measurement of a person’s independence in BADLs before and after intervention and the level of personal care needed for the individual

Oral Prep Phase
Oral manipulation, stretch jaws and tongue, create bolus

Oral Phase
Tongue elevates and moves bolus posteriorly and ends with onset of swallow

Pharyngeal Phase
trigger of swallowing → hyoid and larynx move upward and anteriorly

Esophageal Phase
Swallow Phase, food goes down esophagus and into stomach

Treatment for children with oral hypersensitivity
generalized sensory deep pressure or calming strategies such a slow, linear rocking

Mature head lift and trunk lift
6 months

side to side movement on forearms
6 months

lifting arm to grasp toy
6 months

sitting independently with hands free to play and explore
by 7 months

can sit erect and transition from a prone posture to sitting
8-9 months

3-4 month grasp
automatic with mass finger flexion and no thumb

4-5 month grasp
palmar grasp

6 month grasp
radial palmar grasping

7-12 months grasp
radial digital grasping

voluntary release
7-8 months

purposeful release into container, stacking, and placement
10 months

Visual Foundational Skills
Visual acuity, visual fields, and oculomotor skills. Should be tested first in individuals with acquired brain damage to find underlying cause of visual deficit.

Visual acuity
ability of the eyes to make what is seen sharp and clear

visual fields
allow a person to register the entire visual scene

oculomotor control
control of eye movements and allows the person to move eyes without shifting the head for quick and accurate scanning

Visual cognition
the highest level of the visual-perceptual hierarchy, the ability to mentally manipulate visual information and integrate it with other sensory information to solve problems, formulate plans, and make decisions.

BiVABA
The Brain Injury Assessment Batter for Adults – tests visual foundation skills and visual attention with scanning.

rolls from back to stomach and stomach to back
6 months

begins make believe play
2 years

sorts objects by shape and color
3 years

draws circles and squares
4 years

What should a ct with COPD do if he/she experiences shortness of breath and panics?
1) Rest arms on thighs –> releases the diaphragm, making breathing easier and lessening fear

2) Pursed lip breathing –> can slow breathing and reduce anxiety

What is contraindicated for RA?
1) Increasing AROM

What can be used to initiate compression therapy for a ct 2 weeks post-op skin grafting with full-thickness burns on bilateral anterior/inner thighs?

Also, what could be used during a later stage for compression therapy?

What would be contraindicated?
Spandex bicycle pants

  • Sufficient to apply gentle pressure without causing excessive shear or pressure on the newly adhered graft

Later: Custom-made Jobst garment

Contraindicated: Thigh-high TED stockings (likely to create shear)

Where on the body is Coban™ self-adherent wrap commonly used?
Smaller areas (i.e. fingers)

What is the focus of CBT when applied in Mental Health OT?

  • Social and life skills
  • Focus on self-regulation through observational learning and modeling of skills

What does the psychodynamic approach involve when applied in Mental Health OT?

  • Involves clients projecting feeling that they may be unaware of into art or other activities
  • Activities are used to express emotions

What is the focus of a MOHO approach when applied in Mental Health OT?
Focuses on skilled performance of activities in context

What is the focus of the Dynamic Interactional Model when applied in Mental Health OT?

  • Focuses on multicontextual tasks and environmental conditions that are similar
  • Applying cognitive strategies to facilitate transfer of skills across contexts

In which practice setting is the Outcome and Assessment Information Set (OASIS) completed to help identify the payer for services?
Home Health

Ct. has difficulty seeing gray cars on a rainy day

What does this indicate?
Impaired contrast sensitivity

Ct. needs increased time to identify objects in and along the road and is experiencing eye fatigue

What does this indicate?
Impaired ocular motor skills

(Impaired pursuits, saccades, and scanning may cause the client to stare at a road scene for a longer duration than expected, causing eye strain or fatigue)

Ct has a tendency to drift out of a lane and move too close to other road users or objects

What does this indicate?
Neglect or visual attention deficits

Ct is having difficulty seeing other road users or objects inside a tunnel

What does this indicate?
Visual acuity impairments

What is a positive Gower’s sign and what does it indicate?

  • Asked to get up from sitting on floor –> Child walks the hands up the legs and then moves into a standing position
  • May be indicative of DMD

Abdominal distention in an infant may be a sign of…
Cystic fibrosis

Excessive bruising (not from abuse) may be a sign of…
Hemophilia

A positive Trendelenburg’s sign may be related to…
Hip dysplasia

What do the following symptoms indicate?

Pain, swelling, stiffness, and sudomotor and trophic changes
Complex regional pain syndrome

What do the following symptoms indicate?

Pain, fatigue, and tender trigger points
Fibromyalgia

What do the following symptoms indicate?

Numbness and tingling in the thumb and index and middle fingers
Carpal tunnel syndrome

What do the following symptoms indicate?

Area that is hypersensitive and painful to touch
Neuroma

FIM score of “3” indicates…
Mod A

Name the stages and characteristics of Tuckman’s theory of group development

  • Performing is characterized by group productivity derived from a sense of trust, unity, and supportiveness
  • Forming is characterized by dependence.
  • Storming is characterized by conflict and competition.
  • Norming is characterized by initiation of cohesion

Modifying or adapting task tools is an intervention strategy of …
The Ecology of Human Performance Model

Movement-based activities are a “?” strategy
SI

Strategies to develop mastery are part of the “?” model
Occupational Adaptation Model

Strengthening hand muscles is part of a “?” approach
Biomechanical approach

What is indicated when a ct is moving his wheelchair forward by gripping the front of the pushrim and giving it multiple tiny pushes?
The seat back is to high

–> preventing adequate shoulder extension

What are the principles of Motivational Interviewing?
1) Roll with resistance

2) Express empathy

3) Develop discrepancy

4) Support self-efficacy

A Type III fracture of the radial head requires…
Removal of the fragmented bone and a cast for 3-4 weeks to ensure proper healing and support

The BOT-2 can be used to assess aspects of…
Fine and gross motor functioning that may be difficult as a result of dyspraxia

The SIPT and sensory integration clinical observations provide information related to…
The underlying neurological functions that affect sensory integration

The Sensory Profile can be used to gain an understanding of…
How sensory problems influence a child’s participation in daily activities

Tachydysrhythmia is characterized by…
Fast heart rate of 200-300 beats per minute

  • Fairly common in children with cardiac conditions
  • In extreme cases and when complications are present –> can lead to congestive heart failure

Bradydysrhythmia is characterized by…

Also, a child with bradydysrhythmia usually has..
Heartbeat less than 60 beats per minute

  • An atrioventricular block
  • Pacemaker

What is Donepezil?

Name one side effect.
Cholinergic-modulating drug

  • May improve memory and cognition
  • May reduce negative mood, anxiety, and hallucinations
  • Dizziness is a side effect

A client with low-level (T1) SCI should be..

  • Independent in personal care
  • Only require a few hours of homemaking assistance each day upon discharge

Attendant care 24 hours a day may be indicated for clients with…
Higher level SCI injury (C1-C4).

Attendant care for 12 hours a day may be indicated for clients with…
C5-C6 SCI injury

When dealing with clients who have dementia, what would be the primary role of the OTR in addressing caregiver burden?
Provide recommendations related to home modifications

  • Can benefit clients with dementia and their caregivers by providing a safe environment that prevents unsafe wandering and provides optimal occupational engagement for reduced caregiver burden

Supported employment programs with a “?” perspective are more effective than other vocational approaches
Place-and-train

  • Prevocational training, transitional employment, and sheltered workshops use the classic train-then-place methodology. Because of the emphasis on preparatory skills in these settings, opportunities to transfer skills to actual work settings yield a lower percentage of employment

The National Multiple Sclerosis Society recommends use of the “?” as a fatigue assessment
Modified Fatigue Scale

The Multiple Sclerosis Functional Composite measures…

  • Short-distance walking
  • Hand function
  • Cognition
  • Does not directly assess fatigue

During Type 1 (traumatic stage) CRPS, treatment should focus on…

  • Management of pain and edema along with AROM
  • Most recognized therapeutic intervention for CRPS –> stress loading program

What is contraindicated during Type 1 CRPS?

  • PROM and progressive resistive exercises increase pain and swelling
  • Serial casting may add to joint stiffness
  • Often cold intolerant

A COTA decides that a client has returned to the previous level of function and continued occupational therapy services are no longer needed. Which is the BEST way to discharge this client?
1) COTA alerts the OTR

2) OTR reviews the documentation

3) COTA and OTR discuss the patient’s progress and response to intervention

4) COTA collaborates w/ OTR on a last treatment day

  • OTR judges a client’s need to continue, modify, or stop occupational therapy services, based on information and documentation from the COTA

Where is the decubitus ulcer MOST likely to occur on a client’s body?
Most commonly occurs at a bony prominence that has been compressed against an external surface for a prolonged period

TRICARE provides health coverage for…
The military

The Children’s Health Insurance Program provides coverage for…
Children and families who cannot qualify for Medicaid and cannot afford health insurance

The Federal Employees Health Benefit Program provides coverage for…
Federal employees

What is indicated for De Quervain’s tenosynovitis?

  • Splints typically applied during the acute phase of injury
  • Splint typically prescribed: forearm-based thumb Spica splint

(immobilize the wrist, thumb CMC and MCP joints, placing the abductor pollicis longus and extensor pollicis brevis at rest)

  • Avoid excessive radial and ulnar deviation
  • Use of telephone headset and built-up handled writing implements may relieve or reduce stress on thumb and wrist joints

“?” established the mandate to provide complementary paratransit services under Title II, Part B
The Americans With Disabilities Act of 1990

“?” eliminated unnecessary segregation of people with disabilities and was not specific to transportation
The Supreme Court’s decision in Olmstead v. L. C.

“?” authorized federal surface transportation programs and did not address transportation rights
The Safe, Accountable, Flexible, Efficient Transportation Equity Act: A Legacy for Users (SAFETEA-LU)

“?” expanded civil rights for people with disabilities but did not mandate access to paratransit services
The Rehabilitation Act

Suspicion is characteristic of “?” stage dementia
Early-stage dementia

(Common for clients to think misplaced items have been stolen or that their spouse may be unfaithful)

Pacing, difficulty choosing appropriate clothing, and vulgar or rude language are all common behavioral symptoms of “?” stage dementia
Middle-stage dementia

(Related to loss of impulse control and difficulty communicating needs)

A volar antispasticity hand splint can…
Decrease a soft-tissue contracture once minimal or moderate abnormal tone has been inhibited

A client with a below-the-knee amputation tends to flex the knee when sitting in a wheelchair. Use of a knee extension splint can decrease the knee flexion contracture.

A C bar splint can…
Maintain or improve the first web space affected by a median nerve injury

A knee extension splint can…
Decrease the knee flexion contracture for a ct with a below-the-knee amputation

(Tends to flex the knee when sitting in a wheelchair)

A boney block can only be removed…
Surgically

What strategy might an OTR appropriately recommend to caregivers of a client with Alzheimer’s disease to assist with fall prevention?
Engagement in activity-based interventions & daily structure

(Documented to assist in fall prevention)

The Modified Ashworth Scale is a measure of …
Spasticity

Trigger point evaluation is a measure of…
Localized soft-tissue pain

The FIM is a measure of…
ADL independence

A client with Huntington’s disease presents with decreased motor planning. What is the most effective intervention for an OTR to implement with this cognitive impairment?
Mental imagery

(Effective strategy to improve motor abilities for clients with decreased motor planning)

Healing of the heart muscle takes “?” weeks

MET limitations during this time?
4 to 8 weeks

  • During this time, activities are limited to the 2 to 4 MET range
  • Carrying groceries upstairs requires 6 to 10 METs

Which test assesses a client’s ability to visually select features that differentiate objects?
Selective attention

  • A.K.A. focal attention
  • Ability to disregard irrelevant information or stimuli and attend to what is relevant

The OTR is treating a client who presents with hemiplegia from a CVA. The client’s hand is flaccid. To fabricate a custom resting hand orthosis, in what position should the OTR place the wrist and MCPs?
A resting hand orthosis keeps soft tissue positioned in optimal mobility –> Flexion of the MCPs at 45° to 60°

25° of slight extension would maintain the wrist in a position that would not compromise the median nerve

  • MCPs at 0° of flexion would shorten the collateral ligaments and increase potential for flexion stiffness
  • Wrist positioned in flexion would compromise the median nerve

This condition can lead to heart failure and inadequate oxygenation of the brain
Patent ductus arteriosus (PDA)

  • Heart defect that is common in premature children and people with Down syndrome
  • A defect of the ductus arteriosus results in a lack of constriction –> heart failure and inadequate oxygenation of the brain

This condition can lead to increased blood pressure and rapid heart rate
Tachycardia

This condition can lead to difficulties with feeding and shortness of breath
Supraventricular tachycardia

Job coaching provides…
Vocational evaluation to identify a client’s vocational interests and abilities along with physical and mental tolerances for general job activities

Functional capacity evaluation provides…
Information about a client’s physical abilities to complete a specific job activity

Job site evaluations assess…
The physical demands and layout of a specific job setting

Prework screening is done when…
A client has been offered a job

Desensitization begins with…
The least irritating texture in the least sensitive area to be treated

Desensitization is best used for “?” periods of time
Short periods (3 to 5 minutes five or six times per day

If desensitization is poorly tolerated…
The activity can be combined with use of a TENS unit initially to decrease the client’s perception of pain

An OTR has been working with a client diagnosed with carpal tunnel syndrome. The client’s symptoms have diminished, but now the client reports more pain at the elbow in the median nerve area. When a client reports dual sites of impingement of a single nerve without a history of trauma, what condition is MOST likely the cause?
Double crush syndrome –>

Over time, diminished blood flow to a peripheral nerve can result in serial impingements

Thoracic outlet syndrome involves the…
Shoulder region

Ulnar tunnel syndrome involves what nerve..
Ulnar nerve

Cubital tunnel syndrome involves the…
Ulnar nerve

The Role Checklist assesses…
The value the client places on each of 10 different roles

The Comprehensive Occupational Therapy Evaluation assesses…
Behaviors related to occupational performance

The Performance Assessment of Self-Care Skills measures…
The independence, safety, and adequacy of clients’ ability to care for themselves

Work readiness programs aim to…
Identify a person’s skills and interests to develop his or her readiness for work and to achieve the person’s goals related to work

  • Explores the client’s potential and interests based on current skills and aptitudes
  • Followed by referral to vocational rehabilitation for job placement and job training assistance

Girls with Rett’s syndrome experience…
Decline in functional skills

  • Dependent for meals or donning earrings by early childhood
  • Commonly, no longer be able to walk or talk by late childhood
  • Likely able to express discomfort, even if nonverbal

An OTR is attending an individualized education program (IEP) team planning for a 10-year-old child with autism. Which principle BEST contributes to a good IEP?
To the maximum extent possible, the student must be educated with nondisabled peers

  • IEP must include the student’s strengths as well as needs
  • IEP goal setting is for an entire academic year
  • Transition planning begin at 14 years
  • Transition services begin at 16 years

Medicare Part B covers…
80% of the occupational therapy services in outpatient settings after the yearly deductible

Medicare Part A covers…
Hospital services

An OTR receives a referral for a child who has difficulty with balance and coordination. The OTR wants to determine whether the child has difficulty with visual-receptive functions. Which assessment method would be appropriate to use for this purpose?
Observing how a child’s eyes work together

These methods are ways to assess a child’s visual-cognitive functions…

  • Developmental Test of VMI
  • BOT – (Bruininks-Oseretsky Test of Motor Proficiency)
  • Interview teacher to determine whether child has difficulty w/spelling

An OTR is planning a feeding session with a client with a C5 spinal cord injury (SCI). Which feeding utensil or adaptive equipment would be MOST APPROPRIATE to introduce during the session?
A mobile arm support is best for a client with a C5 injury because the client would most likely show shoulder muscle activity along with biceps and upper trapezius

  • No wrist or finger movement (no tenodesis grasp)

Essential job functions are defined as…
Job duties fundamental to the position the individual holds or desires to hold, as opposed to functions that are marginal

Reasonable accommodations may include…

  • Altered work schedule and duties
  • Facility modifications
  • Purchase of adaptive equipment or assistive technology
  • Modifying or designing a new product

The U.S. Department of Labor is responsible for…

  • Occupational safety
  • Wage and hour standards
  • Unemployment insurance benefits
  • Reemployment services
  • Some economic statistics

Driving: Awareness training is a strategy for…
Clients with neglect or a visual attention deficits

Driving: Adapting side mirrors to deflect bright lights from other vehicles

or

Using a voice-activated GPS to compensate for way-finding problems

are strategies for…
Clients with visual acuity impairments

An OTR is working with a 56-year-old client who has recently undergone a bilateral hip replacement. What musculature is the MOST IMPORTANT to focus on in the client’s strength training?
Emphasis in rehabilitation should be on maintaining motion and increasing strength of surrounding musculature

(e.g. Gluteal muscles)

The hierarchy of functional return involves…

  • Gradual gradation of tasks from gross to fine motor movements
  • Increased resistance
  • Repetition from simple to complex tasks with postural changes to promote neuromuscular reeducation

Which top-down frame of reference emphasizes the three “dimensions of doing” at the participation, performance, and skills level?
MOHO

According to Allen’s Cognitive Screen at Level “?”, the client will be able to wash most distal portions of the body and those that are easily seen but will not always follow the sequence thoroughly. The client may quit before completion and may have difficulty with measuring soaps, lotions, and deodorant.
3.6

At Allen’s Cog Levels “? and ?”, the client will typically recognize the need for a bath and will bathe thoroughly as long as no major problems develop during the bathing process (i.e., lack of soap).
4.0 and 3.8

At Level “?”, the client will typically wash only areas easily seen and reached. The client may wash one area repetitively and may forget to rinse or dry off.
3.4

What MUST be addressed as part of the intervention plan for children who meet eligibility for services under the mandates of the Individualized Family Service Plan (IFSP) component of IDEA, Part C?
Needs of both the child and the family

•Must contain what services (including frequency, intensity, and method of delivery) are needed to meet the needs of the infant or child & family

  • Must include information about natural environments where intervention services will be provided
  • Criteria & procedures to meet the outcomes expected
  • It is not required to include rationale for one-on-one sessions, specific activities that will be used, or reasonable accommodations as part of the written IFSP

Spasticity is indicated when…
A sudden catch or resistance occurs within a quick movement throughout the range of motion for the extremity

Hypertonicity is typically elicited during…
Slow joint movements

Flaccidity indicates…
A lack of muscle tone with no resistance during passive movement

Weakness is assessed through…
Active movement such as moving the extremity against gravity

Four characteristics of successful implementation of a successful work injury prevention program are…
1) Ongoing management support

2) Supervisory support

3) Employee participation

4) Ongoing support and reinforcement of program

The Occupational Self-Assessment is…
A self-report of a client’s perceptions of his or her occupational competence

The Semmes-Weinstein monofilament test measures…
Cutaneous sensation levels but does not measure behavioral responses to sensory experiences

The Kohlman Evaluation of Living Skills evaluates…
a person’s ability to perform basic living skills

To facilitate participation in familiar daily living tasks by clients with dementia, what would be an appropriate INITIAL stage of intervention?
Develop strategies to simplify familiar tasks

  • Task simplification is a primary intervention used with clients with dementia to maintain independence in daily living tasks

A job site analysis looks to define…
The actual demands of the job

  • Involves the use of questionnaire, interviews, observations, and formal assessment completed in the real work environment
  • Many FCEs include a job demand analysis

A hand evaluation is a comprehensive assessment of…

and does NOT assess…
Upper-extremity function

Does not assess the actual material handling skills required for job performance

Situational assessment is used to assess…
A client’s ability to perform the exact same tasks in an environment identical to that of the actual target vocation

Fitness-for-duty evaluation determines…

and screens for…
Whether a worker can perform the essential functions of a specific job

Screens for conditions that may place the worker at risk for injury

  • Takes place only after an employer has offered employment

Early-stage dementia

  • Intact ADL performance
  • Likely to begin having difficulty within the community and with IADLs

Middle Stage Dementia

  • Continue to have skills in habitual ADLs and routines
  • Can maintain conversation
  • Loss of ability to complete most IADLs

Late Stage Dementia
Need assistance for all areas of function, both ADLs and IADLs

  • Limited speech
  • Decreased ambulation
  • Incontinence

An OTR is working with a client with chronic obstructive pulmonary disease (COPD) on a morning ADL routine. Which treatment strategy BEST describes how this client can be independent in showering?
1) Using a shower chair

2) Bathing with warm water from a handheld shower

3) Using pursed lip breathing

4) Exhaling on exertion

  • Hot water adds to humidity and makes breathing more difficult
  • Using a shower chair takes less energy than getting into and out of a tub
  • Cool water may chill the client and require more energy for tasks
  • Leaning forward releases the diaphragm and makes breathing easier

A Ranchos Level II client…
Responds to stimuli only through physiological changes, gross body movement, or vocalization

A Level III client…

  • Inconsistently responds to stimuli
  • Would not be capable of intelligibly responding to simple commands

A Level VII client…

  • Has limited distractibility
  • Capable of carrying out a intelligible conversation despite minimal confusion

A Level V client…

  • Is highly distractible
  • Displays severe memory impairment but can respond to simple commands

K0001

  • Code for a standard chair
  • Most basic, least adjustable wheelchair
  • Meets the minimal standard for Medicare

K0002
Standard one-arm drive wheelchair that can be operated with one hand

K0005

  • Ultra lightweight wheelchair
  • Fully adjustable

K0007
Extra-heavy-duty wheelchair for clients who weigh more than 300 pounds

Progress notes: Documentation content that should be included in work hardening programs…

  • May include pain behaviors, progress to date, meetings held, classes attended, cancellations of appointments, and client’s attempts to problem solve and descriptions of any modifications
  • Should complement initial and discharge summaries and keep the treatment team abreast of progress or problems
  • Daily circuit & schedule sheets are recommended for all clients to foster client responsibility/independence with the program

The ASIA Scale is used to…
Determine baseline sensory and motor control performance for clients with SCI

What is a strategy of the pursed lip breathing technique?

  • Exhaling through pursed lips
  • Exhaling should take twice as long as it does to inhale
  • Inhaling through the mouth is not effective for appropriate oxygen intake
  • Inhaling is done through the nose in breathing techniques (including pursed lip breathing and diaphragmatic breathing) to maximize oxygen intake

The Ecology of Human Performance model aims to focus on…
The interaction of the person, task, and context; therefore, interventions that modify any of these three falls within this model

An OTR is working with a client with C5 spinal cord injury. What is the BEST splinting strategy to use to encourage sensory feedback?
A dorsally based splint will allow for maximal sensory feedback while worn

Dynamic splints

  • Prescribe a low-load resistance
  • Generally cover a large surface of the hand and arm

A claw hand is the result of…
An ulnar nerve injury

  • The 4th and 5th interossei and lumbrical muscles are paralyzed
  • Unopposed extensor digitorum musculotendon unit hyperextends the MCP joints

Low median nerve injury can result in…
Ape hand deformity

High median nerve injury can result in…
Decreased tip pinch

Radial nerve injury can result in…
Wrist drop deformity and decreased finger, thumb, and wrist extension

NMES can be used during an activity to…
Decrease shoulder subluxation post CVA.

Ultrasound and fluidotherapy cannot…
Be used during an activity and do not increase functional control of muscles…………………………………………………………………(Heat increases extensibility soft tissues) ……. (Iontophoresisis used w/fluidotherapy)

TENS can be used during an activity but is…
Typically used for pain and edema control

With lower-extremity burns, providing vascular support before standing…
Decreases blood pooling in the lower extremity –>
decreases pain in standing and ambulation

A 5-year-old client has been referred for fabrication of an orthosis after a flexor tendon repair. Which protocol would be MOST appropriate for this client?
Immobilization orthosis for 3-4 weeks

  • Children who cannot understand or follow a prescribed protocol for motion are best treated initially with an immobilization orthosis to strengthen the repair before movement to reduce the chance of rupture

In addition to corticosteroid injections, what nonoperative treatment is MOST appropriate for de Quervain’s syndrome?
Forearm-based thumb spica splint

  • Thumb and wrist both need to be immobilized

What is de Quervain’s syndrome?
Stenosing tenosynovitis of the abductor pollicis longus and extensor pollicis brevis of the first dorsal compartment

  • Thumb and wrist both need to be immobilized

A wrist cock-up splint to 10° extension would be used for…
Carpal tunnel syndrome

a radial nerve injury post metacarpophalangeal arthroplasty.

A wrist cock-up splint with dynamic extension assist would be used for…
A radial nerve injury post MCP arthroplasty

Overassertive, controlling behaviors during group activities, difficulty engaging in problem solving, and panic over task choices indicate…
Bipolar I disorder

  • Typically consists of more than one combined depressive and manic episode

Aggressive outbursts of behaviors when frustrated with a project, hallucinations, delusions, and disorganized speech indicate….
Schizophrenia

Congenital syphilis may result in…

  • Osteochondritis at the joints
  • Other bone anomalies
  • Dental anomalies
  • Visual and auditory deficits
  • PROM may be painful

Typically, children are able to be independent with toileting (including washing hands and completing clothing management) between the ages of…
4 and 5

Interactive reasoning emphasizes…
The therapist’s effort to understand the client and the client’s life experience

Procedural reasoning is…
Knowing the “how-to” of the therapeutic process

Conditional reasoning involves…
Blending of other forms of reasoning that reflect on interactive and procedural reasoning successes

Narrative reasoning…
Guides other forms of reasoning through an understanding of the client’s personal narratives

“?” mandates that “occupational therapy personnel shall intentionally refrain from actions that cause harm” (AOTA, 2010, p. S19), which includes not only protecting the safety of clients but also, by extension, protecting the public from actions that have a high likelihood of causing harm
Nonmaleficence

The principle of “?” expresses the concept that practitioners have a duty to treat the client according to the client’s desires, within the bounds of accepted standards of care, and to protect the client’s confidential information
Autonomy and confidentiality

“?” is based on the virtues of truthfulness, candor, and honesty; in health care refers to comprehensive, accurate, and objective transmission of information and includes fostering the client’s understanding of such information
Veracity

“?” refers to the fair, equitable, and appropriate distribution of resources
Social justice (also called distributive justice)

What is referred to as “no man’s land”?
Zone II of the flexor tendon system…because excessive scarring makes it difficult to get good results from a repair

Identify which are frequency, duration, and/or intensity:

3 times per week

45-minute sessions

4 weeks

Once a month
3 times per week = frequency

45-minute sessions = intensity

4 weeks = duration

Once a month = frequency

Interoception is the ability to…
Perceive information from inside the body

  • Children with ASD typically have difficulties with toileting because of insufficient ability to perceive the need to urinate and eliminate

What is contraindicated in an acute phase of arthritis?
Resistance applied during manual muscle testing may be harmful to inflamed tissue and joints

A client with chronic inactive rheumatoid arthritis (RA) has been referred for occupational therapy. All the joints in the client’s wrists and hands are affected by the RA, but the only visible sign is the beginning of ulnar drift. What type of splint is BEST to prescribe for this client with the goal of reducing pain at night?
A soft, prefabricated wrist, thumb, and MCP joint support

  • Splinting all the joints of the hand reduces the chances of increased stress on unsupported joints.
  • Studies have suggested that compliance increases with soft splints

Advocating for clients to obtain needed occupational therapy services = ?
Social justice

Respecting the client’s right to refuse occupational therapy services = ?
Relates to autonomy

Supervising other individuals in a responsible and appropriate manner = What ethical principal
Relates to procedural justice

Protecting confidential information of colleagues and fieldwork students = ?
Relates to fidelity

A disability rating considers…
The worker’s impairment and the impact of the impairment on the client’s ability to perform preinjury for any job

  • The unique characteristics of the job

(Impairment rating does not take into account the unique factors relating to worker vocation)

An impairment rating relates to…

  • The percentage of whole-body function and translates to a final monetary settlement for an injured worker
  • Focuses on permanent quantifiable physical loss related to injury when the client is considered to be at the maximum medical endpoint

Fitness-for-duty tests are completed by…
Occupational medicine physicians after injury

Cognitive-behavioral therapy techniques include…

  • Challenging automatic thoughts
  • Reducing cognitive distortions
  • Challenging underlying beliefs and assumptions
  • Visualization
  • Controlling recurrent thoughts
  • Self-monitoring (or controlling) behavior

What is the progression of Duchenne’s muscular dystrophy (DMD)?

  • Progresses quickly
  • Children often need to use a wheelchair by age 9

What is the progression of limb girdle muscular dystrophy?

  • Progresses slowly
  • Children often experience difficulty with muscles around their pelvis and shoulder girdle

What is the progression of facioscapulohumeral muscular dystrophy?

  • Progresses so that the facial muscles have decreased mobility
  • Progresses until children cannot raise their arms above their heads
  • Children eventually have a masklike appearance
  • Usually has an onset before adolescence

For a client with an L2 spinal cord injury, which statement BEST describes the muscle segments below the injury level 1 to 2 months postinjury?
Muscles are spastic

(Because the monosynaptic reflex arc is intact but separated from higher inhibitory influences, muscles below the level of injury generally develop spasticity)

Also…

  • Sensory loss below the level of injury is common
  • Sympathetic functions below the level of injury are hyperactive
  • Although muscles may be initially flaccid for a short period of spinal shock, spasticity regularly develops below the injury

Difficulty with identifying a gray vehicle on a rainy day indicates issues with…
Contrast sensitivity

Difficulty with accurately reading traffic signs or identifying colors on a traffic light indicates issues with…
Color discrimination

Difficulty with reacting quickly when a car pulls out in front of the driver’s vehicle indicates issues with…
Visual processing speed & divided attention deficits

Difficulty with accurately judging the space when pulling into a parking space indicates issues with…
Depth perception

The distinguishing feature of frontotemporal dementia is…
Behavioral changes…

  • Increase in disinhibited behavior
  • Decreased social tact
  • Lack of empathy
  • Lack of interest

The game player…
(traditionally known as a malingerer)

  • Uses symptoms for positive gain
  • Consciously attempts to convince others of the reality of the symptoms

The refugee…
Uses symptoms to avoid an unresolvable conflict

The identified patient…

  • Assumes the role of patient as a lifestyle
  • Sees all accomplishments as being a result of symptoms

The symptom misinterpreter…

  • Responds to physical changes in an extreme manner as a result of unrealistic belief systems or difficulty processing sensory input

What is curb-to-curb service?

  • Requires riders to meet the vehicle at the street
  • Expectation that the traveler can enter and exit the home and the vehicle without assistance

What is Arm-through-arm service?
Rider is offered personal physical assistance to and from the doorway of the home to the door of the vehicle

What is Door-to-door service?
Rider is met at the door and assisted to the door of their destination

What is Demand-responsive service?
Transit system services that are by request as opposed to fixed route

Severe head injuries include GCS scores of…
8 or less

Moderate head injuries include GCS scores of…
9-12

Mild head injuries include GCS scores of…
13-15

The first step in eccentric viewing techniques is…
Increasing the client’s awareness of the central scotoma (blind spot)

Hip precautions for an anterolateral approach are…
1) No external rotation

2) No adduction (crossing legs or feet)

3) No hip extension

  • An abductor wedge will prevent external rotation and adduction of the hips

Glaucoma typically results in…
Peripheral vision loss

Central vision loss often occurs in…
Macular degeneration or cataracts

Fluctuating vision loss often occurs in…
Diabetic retinopathy

Decreased focusing ability is…
An age-associated vision change that relates to the ability to focus on near objects

What is the Print Tool?
Commercially available and standardized way of measuring a child’s ability to produce handwriting

What Level Allens Cognitive is patient determined safe to live independently
4.6

Entry into the early intervention system begins with which of the following?
A. Screening for developmental delay
B. Individualized Family Service Plan
C. Screening for family environmental risk factors
D. Individualized Education Plan (IEP)
A. A family is referred for an early intervention screening at which a developmental therapist or OT screens the child to determine if a full assessment is necessary. (Pediatric Occupational Therapy and Early Intervention by Case-Smith)

In the assessment phase of early intervention, an OT assesses the daily living skills of an infant. In the context of early intervention, the areas that are being assessed are:
A. Feeding and sleeping patterns
B. Play and leisure patterns
C. Motor development patterns
D. Sensory development patterns
A. Daily living skills in infants refer to self-sustaining skills, such as feeding and sleeping. Sleep patterns can be evaluated from the parent’s description of a typical day. Daily living skills also refer to bathing and dressing, but this is as it involves the parents. (Pediatric Occupational Therapy and Early Intervention by Case-Smith)

An assessment team in early intervention has completed an evaluation of a toddler. In compiling the Individualized Family Service Plan, the goals should be determined by the:
A. Service coordinator for the case
B. Therapists from each discipline
C. Reimbursing agency
D. Parents
D. Although therapists suggest goals based on the evaluation process, ultimately a child’s parents decide which goals will be included and which goals are a priority. Goals that relate specifically to the family’s concerns about the care of their child are included in the plan. If therapists have other goals, these can be added with parental consent. (Pediatric Occupational Therapy and Early Intervention by Case-Smith)

In early intervention there are different types of risk factors. A child with Down syndrome is an example of:
A. Biological risk
B. Environmental risk
C. Established risk
D. Recurring risk
C. Children with known chromosomal, structural, or metabolic defects are classified as having established risk. (Pediatric Occupational Therapy and Early Intervention by Case-Smith)

The parent of a child that is being treated in therapy describes how the child covers their ears when riding in the car with the windows down. The parent does not understand why the child persists in this behavior. The OT explains that this behavior could be the result of:
A. Sensory defensiveness
B. Gravitational insecurity
C. Underresponsiveness
D. Aversion to movement
A. A child experiencing sensory defensiveness has a tendency to respond negatively to sensation that is considered by most people to be noninvasive or nonirritating. This frequently includes hyperresponsiveness to light or unexpected touches, high-frequency noises, certain visual stimulation, or certain smells and tastes. (Sensory Integration: Theory and Practice by Bundy)

An OT is using the Peabody Developmental Motor Scales to evaluate a child. The therapist is assessing the child’s:
A. Performance of tasks that support school participation
B. Visual perception skills in community settings
C. Gross and fine motor skills
D. Performance in everyday tasks
C. The Peabody assesses a child’s abilities with gross and fine motor skills. (Occupational Therapy for Children by Case-Smith)

A patient diagnosed with insulin dependent diabetes mellitus is referred to occupational therapy for splinting. A primary area that must be assessed before prescribing a splint is:
A. Edema
B. Sensation
C. Pain
D. Fine motor manipulation
B. Persons with long-standing diabetes frequently have increased incidences of other conditions, such as peripheral neuropathies. Therefore, a sensory evaluation is necessary to determine if sensation is diminished. A person with diminished sensation secondary to peripheral neuropathy may not be able to perceive or gauge pressure when wearing a splint. This can lead to skin breakdown. (Introduction to Splinting: A Clinical Reasoning & Problem Solving Process by Coppard and Lohman)

An OT is asked to administer a test to a child and compare the assessment results or scores to the sample population of children that have similar characteristics as this child. The BEST type of evaluation to administer would be:
A. Criterion-referenced test
B. Norm-referenced test
C. Skilled observation
D. Checklist
B. A norm-referenced test is developed by giving the test in question to a large number of children, usually several hundred or more. This group is the more normative group and norms or averages are derived from this sample. When a norm-referenced test is administered, the performance of the child being tested is compared to the normative sample. (Occupational Therapy for Children by Case-Smith)

During an evaluation, the OT must determine a child’s exact chronological age. The child was born on March 6, 2003 and the testing date is July 12, 2006. The child’s chronological age is:
A. 4 years, 6 months, 5 days
B. 3 years, 2 months, 6 days
C. 4 years, 5 months, 6 days
D. 3 years, 4 months, 6 days
D. The day, month, and year of the child’s birth date is subtracted from the date of testing. (Occupational Therapy for Children by Case-Smith)

An OT has to calculate the corrected age for a child that was born prematurely. The child had a due date of September 20, 2005 and their birth date was June 12, 2005. The child was born 3 months, 8 days premature and is currently 1 year, 1 month, 25 days old. The therapist determines the corrected age is:
A. 10 months, 17 days
B. 12 months, 2 days
C. 9 months, 8 days
D. 7 months, 10 days
A. Corrected age is used for children who were born prematurely to “correct” for the number of weeks they were born before the due date. Generally the age is corrected until the child turns 2 years old. Many practitioners consider 36 to 37 weeks and above to be full-term gestation. Children with a gestation period of 36 weeks and above do not receive a corrected age. Subtract the birth date from the due date to yield the exact measurement of prematurity, and to calculate the corrected age, subtract the amount of prematurity from the chronological age. (Occupational Therapy for Children by Case-Smith)

A 3-year-old child has been referred for early intervention. In the discussion about intervention with the family, the team should be sure to:
A. Use lay terminology to describe the early intervention process
B. Explain conditions in detailed technical and medical terms
C. Discourage parents asking questions
D. Ignore parents’ feedback and ideas on intervention
A. The team dealing with the case should use lay terminology to describe the early intervention process and repeat information to make sure that the parents understand. They should also welcome parental feedback and incorporate the parents’ ideas into a suggested activity. Professionals should encourage parents to ask questions and repeat information when necessary. Because the language used by health care professionals is often technical and medically related, families can easily misunderstand its meaning. OTs and other team members must make a concerted effort to use lay terminology to describe function, rather than neurophysiologic components. (Occupational Therapy for Children by Case-Smith)

A 6-year-old is interested in learning to roller skate. However, after the initial few minutes of practice the child does not continue with it and appears to lack the will to follow up what was started. This behavior is typical of Erik Eriksson’s psychosocial development stage that deals with:
A. Basic trust versus mistrust stage
B. Autonomy versus doubt and shame stage
C. Self-identity versus role diffusion stage
D. Security versus instability
B. This stage is characterized by holding on and letting go and is exemplified by the crisis that occurs through the toilet-training process. This stage brings independent movement away from the parents, enabling the child to explore the environment. Parents must provide opportunities for the child to make choices and develop a sense of self-controlled will. (Occupational Therapy for Children by Case-Smith)

Which one of the four components of the adaptation process pertains to reception of sensory stimuli from internal and external environments?
A. Assimilation
B. Accommodation
C. Association
D. Differentiation
A. Assimilation is the reception of sensory stimuli from internal and external environments. Accommodation is the motor response to these stimuli. Association is the organized process of relating current sensory information with the current motor response and then relating this relationship to past responses. Differentiation is the process of identifying the specific elements in a situation that are useful and relevant to another situation to refine the responsive pattern. (Occupational Therapy for Children by Case-Smith)

A 4-year-old has been diagnosed with mental retardation. A characteristic that is likely to occur with impaired intellectual ability is:
A. Acceptable social skills
B. Impairment of occupational performance areas
C. Development of bizarre attachment to unusual objects
D. Poor eye contact
B. A child diagnosed with mental retardation will show significantly impaired intellectual ability, behavioral deficits, and impairmnet of the skills required for independence in occupational performance areas (i.e., age-appropriate play, dressing, and communication). (Occupational Therapy for Children by Case-Smith)

A 3-year-old has accidentally ingested lead while playing near ceramic tiles that the family bought to renovate their house. A system that an OT working in pediatric acute care would note to be affected by lead poisoning would be:
A. Vocal
B. Circulatory
C. Digestive
D. Cardiac
B. In lead poisoning, lead affects the circulatory system by severely limiting the body’s ability to synthesize heme, leading to the accumulation of alternate metabolites in the body and, ultimately, anemia. The most significant and irreversible damage occurs in the nervous system. Fluid builds up in the brain, and intracranial pressure can reach life-threatening levels. Cortical atrophy and lead encephalitis can lead to mental retardation, paralysis, blindness, and deafness. (Occupational Therapy for Children by Case-Smith)

A child with cerebral palsy shows significant impairment in the function of the lower extremities with mild involvement of the upper extremities. The classification of cerebral palsy for this child would be:
A. Hemiplegia
B. Tetraplegia
C. Choreoathetosis
D. Diplegia
D. In diplegia, the lower extremities are impaired significantly, with only mild involvement of the upper extremities. Hemiplegia involves the impairment of upper and lower limbs on one side of the body, and tetraplegia or quadriplegia means that all four extremities are affected. Therefore, diplegia is the possible classification of the child’s condition. (Occupational Therapy for Children by Case-Smith)

A 7-year-old child is diagnosed with attention deficit hyperactivity disorder (ADHD). An etiology for ADHD would be:
A. Environmental factors
B. Visual and auditory stimulation
C. Food allergies and food additives
D. Neurochemical imbalances
D. Although researchers are still struggling to find conclusive evidence for the etiology of ADHD, studies have demonstrated that ADHD runs in families and three genes have been found to be related to ADHD implicating genetic factors. Imaging in individuals with ADHD has shown decreased activity in the frontal parietal lobes, which inhibit impulsiveness, pointing to neurologic factors. Medications that influence neurotransmitter function are effective in treatment of ADHD, indicating neurochemical imbalances. (Occupational Therapy for Children by Case-Smith)

A 5-year-old child has been diagnosed with a pervasive disorder that affects both the neurologic and motor behavioral functions. The diagnosis that MOST closely relates to the child’s condition is:
A. Respiratory distress syndrome
B. Tourette’s syndrome
C. Asperger’s syndrome
D. Learning disabilities
B. Tourette’s syndrome is a pervasive disorder that affects neurologic and motor behavioral function, especially motor tics and vocalizations. Therefore, it is likely that the child has Tourette’s syndrome. (Occupational Therapy for Children by Case-Smith)

A 5-year-old child with Down syndrome shows significant loss of weight, high fever, and paleness, and is diagnosed with acute lymphoid leukemia. In which of the following phases of leukemia will he be administered chemotherapy to treat small deposits of cells that remain after remission?
A. Phase I – Induction therapy
B. Phase II – Central nervous system prophylaxis
C. Phase III – Intensification and consolidation
D. Phase IV – Maintenance or continuation therapy
C. Chemotherapy is administered in Phase III – the intensification and consolidation phase- to remove small deposits of cells that remain after remission. (Occupational Therapy for Children by Case-Smith)

A 5-year-old child presents with difficulty climbing stairs, rising from a sitting or lying position, and demonstrates progressive fatigue caused by muscle weakness. The OT might suspect a problem with:
A. Duchenne’s muscular dystrophy
B. Limb-girdle muscular dystrophy
C. Facioscapulohumeral muscular dystrophy
D. A congenital muscular dystrophy
A. Duchenne’s muscular dystrophy is an X-linked recessive disorder caused by the deficiency in the production of dystrophin. Dystrophin is a component of the plasma membrane of the muscle fibers, the deficiency of which causes the muscle to degenerate and malfunction. The calf muscles are enlarged because of fibrosis and proliferation of the adipose tissue, which causes muscle weakness. Therefore, the child’s condition could be diagnosed as Duchenne’s muscular dystrophy. (Occupational Therapy for Children by Case-Smith)

The behavioral characteristics of autism, a pervasive developmental disorder (PDD), can be classified into four subclusters of disturbances. Of these disturbances, which is MOST closely related to prognosis?
A. Disturbances in communication
B. Disturbances in behaviors
C. Disturbances in social interactions
D. Disturbances of sensory and perceptual processing
A. Disturbances in communication can range from mild (slight articulation impairment) to severe (muteness); thus, the more mild the communication deficit, the more likely the child will develop sufficient communication to function as an adult. (Occupational Therapy for Children by Case-Smith)

A 3-year-old child is diagnosed with cerebral palsy and failure to thrive, and reportedly has difficulty with drooling, chewing, and swallowing. The OT should address:
A. Oral motor deficits
B. Motor sensory deficits
C. Self-feeding
D. Vestibular input
A. Oral motor deficits relate to issues with the oral motor musculature around the mouth. OTs are qualified to address deficits related to swallowing or dysphagia, chewing, and drooling. (Occupational Therapy for Children by Case-Smith)

A kindergarten teacher observed that a 5-year-old child does not participate in play with other children and avoids movement activities on the playground. A screening test you would recommend for this child to determine if there is a need for a more comprehensive examination is:
A. Ages & Stages Questionnaires
B. The First STEP
C. Short Sensory Profile
D. Denver Developmental Screening Test-II (Denver-II)
C. The teacher has already identified the child as being at risk for developmental or functional deficits. Therefore, it is appropriate to use a type II assessment, which is the Short Sensory Profile. This caregiver questionnaire measures the frequency of behaviors related to sensory processing, modulation, and emotional responsivity to sensory input in children 3 to 12 years of age. The Ages & Stages Questionnaires, First STEP, and the Denver-II pertain to the early screening to identify children at risk for disabilities. (Occupational Therapy for Children by Case-Smith)

A scenario in which an OT would use the School Function Assessment examination is:
A. To measure the student’s schoolwork task performance in the classroom and provide information for effective programs and consultation in the school setting
B. To assess what the child “did do” and what the child “could do” to help determine the effect of the child’s physical disability on engagement in everyday occupations
C. For screening children to determine whether they warrant further, more comprehensive evaluation
D. To measure children’s ability to participate in the academic and social aspects of the school environment
D. The School Function Assessment is a judgment-based questionnaire designed to measure a student’s performance of functional tasks that support his or her participation in the academic and social aspects of an elementary school program (kindergarten through grade 6). Three scales evaluate the student’s level of participation, the type and amount of task supports needed, and his or her activity performance on specific school tasks. (Occupational Therapy for Children by Case-Smith)

An occupational therapy assistant (OTA) works with an OT in an early intervention program at a local school. The portion of the evaluation that the OT can assign the OTA to perform is:
A. Selecting evaluation methods and measures
B. Interpreting and analyzing assessment data
C. Administering some of the assessments
D. Documenting some of the goals
C. According to the Standards of Practice for Occupational Therapy, an OTA would perform functions such as administering some of the assessments and documenting some of the results. Selecting evaluation methods and measures, interpreting and analyzing assessment data, and writing goals are function of the OT. (Occupational Therapy for Children by Case-Smith)

Leave a Comment

Scroll to Top